Pharm MT3

Réussis tes devoirs et examens dès maintenant avec Quizwiz!

1. Which statement about the mechanisms of action of antiviral drugs is accurate? (A) Acyclovir has no requirement for activation by phosphorylation (B) An increase in activity of host cell ribonucleases that degrade viral mRNA is one of the antiviral actions of interferon-α (C) Ganciclovir inhibits viral DNA polymerase but does not cause chain termination (D) The initial step in activation of foscarnet in HSVinfected cells is its phosphorylation by thymidine kinase (E) The reverse transcriptase of HIV is 30-50 times more sensitive to inhibition by fosamprenavir than host cell DNA polymerases

1. Acyclovir is activated by host cell kinases. Like acyclovir, ganciclovir inhibits viral DNA polymerase and causes chain termination. However, foscarnet inhibits viral DNA polymerase without requiring bioactivation. Fosamprenavir is the prodrug of amprenavir, an inhibitor of HIV protease; it has no significant effect on reverse transcriptase. The answer is B.

1. The primary reason for the use of drug combinations in the treatment of tuberculosis is to (A) Delay or prevent the emergence of resistance (B) Ensure patient compliance with the drug regimen (C) Increase antimycobacterial activity synergistically (D) Provide prophylaxis against other bacterial infections (E) Reduce the incidence of adverse effects

1. Although it is sometimes possible to achieve synergistic effects against mycobacteria with drug combinations, the primary reason for their use is to delay the emergence of resistance. The answer is A.

1. Regarding the mechanism of action of aminoglycosides, the drugs (A) Are bacteriostatic (B) Bind to the 50S ribosomal subunit (C) Cause misreading of the code on the mRNA template (D) Inhibit peptidyl transferase (E) Stabilize polysomes

1. Aminoglycosides are bactericidal inhibitors of protein synthesis binding to specific components of the 30S ribosomal subunit. Their actions include block of the formation of the initiation complex, miscoding, and polysomal breakup. Peptidyl transferase is inhibited by chloramphenicol, not aminoglycosides. The answer is C.

1. A 2-year-old child is brought to the hospital after ingesting pills that a parent had used for bacterial dysentery when traveling outside the United States. The child has been vomiting for more than 24 h and has had diarrhea with green stools. He is now lethargic with an ashen color. Other signs and symptoms include hypothermia, hypotension, and abdominal distention. The drug most likely to be the cause of this problem is (A) Ampicillin (B) Chloramphenicol (C) Clindamycin (D) Doxycycline (E) Erythromycin

1. Chloramphenicol is commonly used outside the United States for treatment of bacillary dysentery. The drug causes a dose-dependent (reversible) suppression of erythropoiesis. Although the gray baby syndrome was initially described in neonates, a similar syndrome has occurred with overdosage of chloramphenicol in older children and adults, especially those with hepatic dysfunction. The answer is B.

1. Which of the following is an estrogen that is used in most combined hormonal contraceptives? (A) Clomiphene (B) Estrone (C) Ethinyl estradiol (D) Diethylstilbestrol (DES) (E) Norgestrel

1. Ethinyl estradiol, a synthetic estrogen with good bioavailability, is the estrogenic component of most combined oral contraceptives, the transdermal contraceptive, and the vaginal ring contraceptive. The answer is C.

Questions 1-3. A 32-year-old woman underwent segmental mastectomy for a breast tumor of 3 cm diameter. Lymph node sampling revealed 2 involved nodes. Because chemotherapy is of established value in her situation, she underwent postoperative treatment with antineoplastic drugs. The regimen consisted of doxorubicin followed by cyclophosphamide/methotrexate/fluorouracil. Adjunctive drugs included tamoxifen because the tumor cells were hormone receptor-positive. 1. Which of the following best describes the mechanism of anticancer action of cellular metabolites of fluorouracil? (A) Cross-linking of double-stranded DNA (B) Inhibition of DNA-dependent RNA synthesis (C) Interference with the activity of topoisomerases I (D) Irreversible inhibition of thymidylate synthase (E) Selective inhibition of DNA polymerases

1. Fluorouracil (5-FU) undergoes metabolism to form 5-fluoro- 2′-deoxyuridine 5′-phosphate (5-dUMP). This metabolite forms a covalently bound ternary complex with thymidylate synthase and its coenzyme N-methylenetetrahydrofolate. The synthesis of thymine nucleotides is blocked, DNA synthesis is inhibited, and a "thymineless death" of cells results. The answer is D.

1. Which of the following is a pharmacologic effect of exogenous glucocorticoids? (A) Increased muscle mass (B) Hypoglycemia (C) Inhibition of leukotriene synthesis (D) Improved wound healing (E) Increased excretion of salt and water

1. Glucocorticoids inhibit the production of both leukotrienes and prostaglandins. This is a key component of their antiinflammatory action. The answer is C.

1. A missionary from the United States is sent to work in a geographic region of a Central American country where Onchocerca volvulus is endemic. Infections resulting from this tissue nematode (onchocerciasis) are a cause of "river blindness," because microfilariae migrate through subcutaneous tissues and concentrate in the eyes. Which drug should be used prophylactically to prevent onchocerciasis? (A) Albendazole (B) Diethylcarbamazine (C) Ivermectin (D) Oxamniquine (E) Suramin

1. Ivermectin prevents onchocerciasis and is the drug of choice in the individual and mass treatment of the disease. The only other drugs listed with any activity against Onchocerca volvulus are suramin and diethylcarbamazine. However, they are no longer recommended for onchocerciasis because they are less effective and more toxic than ivermectin. The answer is C.

1. The primary mechanism of antibacterial action of the penicillins involves inhibition of (A) Beta-lactamases (B) N-acetylmuramic acid synthesis (C) Peptidoglycan cross-linking (D) Synthesis of cell membranes (E) Transglycosylation

1. Penicillins (and cephalosporins) bind to PBPs acting at the transpeptidation stage of cell wall synthesis (the final step) to inhibit peptidoglycan cross-linking. The beta-lactam antibiotics also activate autolysins, which break down the bacterial cell wall. Synthesis of N-acetylmuramic acid is inhibited by fosfomycin. Vancomycin inhibits transglycolase preventing elongation of peptidoglycan chains. The answer is C.

1. Which statement about antiprotozoal drugs is accurate? (A) Chloroquine is an inhibitor of plasmodial dihydrofolate reductase (B) Mefloquine destroys secondary exoerythrocytic schizonts (C) Primaquine is a blood schizonticide and does not affect secondary tissue schizonts (D) Proguanil complexes with double-stranded DNA-blocking replication (E) Trimethoprim-sulfamethoxazole is the drug of choice for Pneumocystis jiroveci pneumonia

1. Proguanil (not chloroquine) is an inhibitor of dihydrofolate reductase. Primaquine (not mefloquine) is the drug that destroys secondary exoerythrocytic schizonts. TMP-SMZ is the drug of choice for Pneumocystis jiroveci pneumonia. The answer is E.

1. Which of the following is a drug that is purified from the urine of postmenopausal women and used to promote spermatogenesis in infertile men? (A) Desmopressin (B) Gonadorelin (C) Goserelin (D) Somatropin (E) Urofollitropin

1. Spermatogenesis in males requires the action of FSH and LH. Urofollitropin, which is purified from the urine of postmenopausal women, is used clinically to provide FSH activity. The answer is E.

Questions 1-3. A 24-year-old woman was found to have mild hyperthyroidism due to Graves' disease. She appears to be in good health otherwise. 1. In Graves' disease, the cause of the hyperthyroidism is the production of an antibody that does which of the following? (A) Activates the pituitary thyrotropin-releasing hormone (TRH) receptor and stimulates TSH release (B) Activates the thyroid gland TSH receptor and stimulates thyroid hormone synthesis and release (C) Activates thyroid hormone receptors in peripheral tissues (D) Binds to thyroid gland thyroglobulin and accelerates its proteolysis and the release of its supply of T4 and T3 (E) Binds to thyroid-binding globulin (TBG) and displaces bound T4 and T3

1. The antibodies produced in Graves' disease activate thyroid gland TSH receptors. Their effects mimic those of TSH. The answer is B.

1. Interactions between this drug and cell membrane components can result in the formation of pores lined by hydrophilic groups present in the drug molecule. (A) Amphotericin B (B) Flucytosine (C) Griseofulvin (D) Itraconazole (E) Terbinafine

1. The polyene antifungal drugs are amphipathic molecules that can interact with ergosterol in fungal cell membranes to form artificial pores. In these structures, the lipophilic groups on the drug molecule are arranged on the outside of the pore, and the hydrophilic regions are located on the inside. The fungicidal action of amphotericin B and nystatin derives from this interaction, which results in leakage of intracellular constituents. The answer is A.

1. Trimethoprim-sulfamethoxazole is established to be effective against which of the following opportunistic infections in the AIDS patient? (A) Cryptococcal meningitis (B) Disseminated herpes simplex (C) Oral candidiasis (D) Toxoplasmosis (E) Tuberculosis

1. Trimethoprim-sulfamethoxazole is not effective in the treatment of infections caused by viruses, fungi, or mycobacteria. However, the drug combination is active against certain protozoans, including Toxoplasma, and can be used for both prevention and treatment of toxoplasmosis in the AIDS patient. The answer is D.

10. More than 90% of this drug is excreted in the urine in intact form. Because its urinary solubility is low, patients should be well hydrated to prevent nephrotoxicity. Which drug is described? (A) Acyclovir (B) Efavirenz (C) Indinavir (D) Trifluridine (E) Zidovudine

10. Acyclovir is eliminated in the urine by glomerular filtration and by active tubular secretion, which is inhibited by probenecid. Nephrotoxic effects, including hematuria and crystalluria, are enhanced in patients who are dehydrated or who have preexisting renal dysfunction. Adequate hydration is equally important in the case of indinavir because it causes nephrolithiasis. However, more than 80% of a dose of indinavir is eliminated via hepatic metabolism. Trifluridine is used topically to treat herpes keratoconjunctivitis. The answer is A.

10. A malnourished 12-year-old child who lives in a rural area of the southern United States presents with weakness, fever, cough, abdominal pain, and eosinophilia. His mother tells you that she has seen long, thin worms in the child's stools, sometimes with blood. A presumptive diagnosis of ascariasis is confirmed by the presence of the ova of A lumbricoides in the stools. However, microscopy also reveals that the stools contain the eggs of Necator americanus. The drug most likely to be effective in the treatment of this child is (A) Albendazole (B) Diethylcarbamazine (C) Ivermectin (D) Niclosamide (E) Praziquantel

10. Albendazole is effective against both nematodes causing infection in this child. Mebendazole and pyrantel pamoate (not listed in this question) are also primary drugs for the treatment of combined infections due to hookworm and roundworm. The answer is A.

10. A 52-year-old postmenopausal patient has evidence of low bone mineral density. She and her physician are considering therapy with raloxifene or a combination of conjugated estrogens and medroxyprogesterone acetate. Which of the following patient characteristics is most likely to lead them to select raloxifene? (A) Previous hysterectomy (B) Recurrent vaginitis (C) Rheumatoid arthritis (D) Strong family history of breast cancer (E) Troublesome hot flushes

10. Conjugated estrogens and raloxifene both improve bone mineral density and protect against osteoporosis. The 2 advantages of raloxifene over full estrogen receptor agonists are that raloxifene has antagonist effects in breast tissue and lacks an agonistic effect in endometrium. If a patient's uterus was removed by surgery, the difference in the endometrial effect is moot. In patients with a strong family history of breast cancer, raloxifene may be a better choice than a full estrogen agonist because it will not further increase the woman's risk of breast cancer and may even lower her risk. The answer is D.

10. All the following agents have been used in drug regimens for the treatment of breast carcinoma. Which one has specific activity in a subset of female breast cancers? (A) Cyclophosphamide (B) Doxorubicin (C) Fluoxymesterone (D) Methotrexate (E) Trastuzumab

10. Each of the drugs listed has been used in drug regimens for breast cancer, but only trastuzumab has specificity in its actions. The drug is a monoclonal antibody to a surface protein in breast cancer cells that overexpress the HER-2 protein. Consequently, trastuzumab has value in a specific subset of breast cancers. The answer is E.

10. Which statement about "once-daily" dosing with aminoglycosides is not accurate? (A) Convenient for outpatient therapy (B) Dosage adjustment is less important in renal insufficiency (C) Less nursing time is required for drug administration (D) Often less toxic than conventional (multiple) dosing regimens (E) Underdosing is less of a problem

10. In "once-daily dosing" with aminoglycosides, the selection of an appropriate dose is particularly critical in patients with renal insufficiency. The aminoglycosides are eliminated by the kidney in proportion to creatinine clearance. Knowledge of the degree of insufficiency, based on plasma creatinine (or BUN), is essential for estimation of the appropriate single daily dose of an aminoglycoside. The answer is B.

10. Regarding the clinical use of liposomal formulations of amphotericin B, which statement is accurate? (A) Amphotericin B affinity for these lipids is greater than affinity for ergosterol (B) Less expensive to use than conventional amphotericin B (C) More effective in fungal infections because they increase tissue uptake of amphotericin B (D) They decrease the nephrotoxicity of amphotericin B (E) They have a wider spectrum of antifungal activity than conventional formulations of amphotericin B

10. Liposomal formulations of amphotericin B result in decreased accumulation of the drug in tissues, including the kidney. As a result, nephrotoxicity is decreased. With some lipid formulations, infusion-related toxicity may also be reduced. Lipid formulations do not have a wider antifungal spectrum; their daily cost ranges from 10 to 40 times more than the conventional formulation of amphotericin B. The answer is D.

10. Metronidazole is not effective in the treatment of (A) Amebiasis (B) Infections due to Bacteroides fragilis (C) Infections due to Pneumocystis jiroveci (D) Pseudomembranous colitis (E) Trichomoniasis

10. Metronidazole is the drug of first choice for all of the conditions listed except pneumocystosis. The answer is C.

10. Which of the following is a drug that, in high doses, blocks the glucocorticoid receptor? (A) Aminoglutethimide (B) Beclomethasone (C) Ketoconazole (D) Mifepristone (E) Spironolactone

10. Mifepristone is a competitive antagonist of glucocorticoid and progesterone receptors. Ketoconazole and aminoglutethimide also antagonize corticosteroids; however, they act by inhibiting steroid hormone synthesis. The answer is D.

10. Risk factors for multidrug-resistant tuberculosis include (A) A history of treatment of tuberculosis without rifampin (B) Recent immigration from Asia and living in an area of over 4% isoniazid resistance (C) Recent immigration from Latin America (D) Residence in regions where isoniazid resistance is known to exceed 4% (E) All of the above

10. Multidrug-resistant tuberculosis (MDR-TB) is defined as resistance to 2 or more drugs. All the risk factors are relevent. In the case of resistance to both INH and rifampin, initial regimens still include both drugs plus ethambutol, pyrazinamide, streptomycin (or other aminoglycoside), and a fluoroquinolone. Continuation therapy should include at least 3 drugs shown to be active in vitro against the infecting strain. The appropriate duration of therapy has not been established. The answer is E.

10. A 7-year-old boy underwent successful chemotherapy and cranial radiation for treatment of acute lymphocytic leukemia. One month after the completion of therapy, the patient presented with excessive thirst and urination plus hypernatremia. Laboratory testing revealed pituitary diabetes insipidus. To correct these problems, this patient is likely to be treated with which of the following? (A) Corticotropin (B) Desmopressin (C) hCG (D) Menotropins (E) Thyrotropin

10. Pituitary diabetes insipidus results from deficiency in vasopressin. It is treated with desmopressin, a peptide agonist of vasopressin V2 receptors. The answer is B.

10. Concerning quinupristin-dalfopristin, which statement is accurate? (A) Active in treatment of infections caused by E faecalis (B) Bacteriostatic (C) Hepatotoxicity has led to FDA drug alerts (D) Induce formation of hepatic drug-metabolizing enzymes (E) Used in management of infections caused by multidrugresistant streptococci

10. Quinupristin-dalfopristin is bactericidal against many drug-resistant gram-positive cocci, including multidrug-resistant streptococci, MRSA, and vancomycin-resistant enterococci. The streptogramins have activity against E faecium (not E fecalis). The drugs are potent inhibitors of CYP3A4 and interfere with the metabolism of many other drugs. The streptogramins are not hepatotoxic. The answer is E.

10. Drug that produces a permanent reduction in thyroid activity (A) 131I (B) Amiodarone (C) Propranolol (D) Propylthiouracil (E) Triiodothyronine

10. Radioactive iodine is the only medical therapy that produces a permanent reduction of thyroid activity. The answer is A.

10. Supplementary folinic acid may prevent anemia in folatedeficient persons who use this drug; it is a weak base achieving tissue levels similar to those in plasma (A) Ciprofloxacin (B) Moxifloxacin (C) Sulfacetamide (D) Sulfamethoxazole (E) Trimethoprim

10. Trimethoprim is the only weak base listed (fluoroquinolones and sulfonamides are acidic compounds), and its high lipid solubility at blood pH allows penetration of the drug into prostatic and vaginal fluid to reach levels similar to those in plasma. Leukopenia and thrombocytopenia may occur in folate deficiency when the drug is used alone or in combination with sulfamethoxazole. Fluoroquinolones do not exacerbate symptoms of folic acid deficiency. The answer is E.

10. Which statement about vancomycin is accurate? (A) Active against methicillin-resistant staphylococci (B) Bacteriostatic (C) Binds to penicillin-binding proteins (PBPs) (D) Hepatic metabolism (E) Oral bioavailability

10. Vancomycin is a bactericidal glycoprotein. It inhibits cell wall synthesis but does not bind to PBPs and is not susceptible to beta-lactamases. Vancomycin is not absorbed after oral administration and is used by this route in the treatment of colitis caused by C difficile and staphylococci. It undergoes renal elimination. Vancomycin is commonly considered the drug of first choice for parenteral use against methicillinresistant staphylococci. The answer is A.

11. If allopurinol is used adjunctively in cancer chemotherapy to offset hyperuricemia, the dosage of this anticancer drug should be reduced to 25% of normal. (A) Bleomycin (B) Cytarabine (C) Dacarbazine (D) Doxorubicin (E) Etoposide (F) Flutamide (G) Fluorouracil (H) Leuprolide (I) Mechlorethamine (J) Mercaptopurine (K) Methotrexate (L) Paclitaxel (M) Procarbazine (N) Tamoxifen (O) Vincristine

11. Allopurinol, a xanthine oxidase inhibitor, is given to control the hyperuricemia that occurs as a result of large cell kills in the successful drug therapy of malignant diseases. The antimetabolite mercaptopurine is metabolized by xanthine oxidase and, in the presence of an inhibitor of this enzyme (eg, allopurinol), toxic levels of the drug may be reached rapidly. The answer is J.

12. This drug is used in combination therapy for testicular carcinoma. It is a CCS drug that acts in the late S and early G0 phases of the tumor cell cycle via interactions with topoisomerase II. (A) Bleomycin (B) Cytarabine (C) Dacarbazine (D) Doxorubicin (E) Etoposide (F) Flutamide (G) Fluorouracil (H) Leuprolide (I) Mechlorethamine (J) Mercaptopurine (K) Methotrexate (L) Paclitaxel (M) Procarbazine (N) Tamoxifen (O) Vincristine

12. Bleomycin, etoposide, and vinblastine are all CCS drugs used for the treatment of testicular carcinoma. Bleomycin is an antibiotic, not a plant alkaloid. Vinblastine is a mitotic spindle poison that acts in the M phase of the cell cycle. The answer is E.

13. This antimetabolite inhibits DNA polymerase and is one of the most active drugs in leukemias. Although myelosuppression is dose limiting, the drug may also cause cerebellar dysfunction, including ataxia and dysarthria. (A) Bleomycin (B) Cytarabine (C) Dacarbazine (D) Doxorubicin (E) Etoposide (F) Flutamide (G) Fluorouracil (H) Leuprolide (I) Mechlorethamine (J) Mercaptopurine (K) Methotrexate (L) Paclitaxel (M) Procarbazine (N) Tamoxifen (O) Vincristine

13. The pyrimidine antimetabolite cytarabine (Ara-C) is commonly used in drug regimens for the acute leukemias. Cytarabine is dose-limited by hematotoxicity. Cerebellar dysfunction may also occur with Ara-C, especially if the drug is used at high doses. The answer is B.

2. Which statement about fluconazole is accurate? (A) Does not penetrate the blood-brain barrier (B) Has the least effect of all azoles on hepatic drug metabolism (C) Is an inducer of hepatic drug-metabolizing enzymes (D) Is highly effective in the treatment of aspergillosis (E) Oral bioavailability is less than that of ketoconazole

2. The azoles with activity against Aspergillus are itraconazole and voriconazole. Fluconazole is the best absorbed member of the azole group by the oral route and the only one that readily penetrates into cerebrospinal fluid. Although fluconazole may inhibit the metabolism of some drugs, it has the least effect of all azoles on hepatic microsomal drug-metabolizing enzymes. The answer is B.

Questions 1-3. A 32-year-old woman underwent segmental mastectomy for a breast tumor of 3 cm diameter. Lymph node sampling revealed 2 involved nodes. Because chemotherapy is of established value in her situation, she underwent postoperative treatment with antineoplastic drugs. The regimen consisted of doxorubicin followed by cyclophosphamide/methotrexate/fluorouracil. Adjunctive drugs included tamoxifen because the tumor cells were hormone receptor-positive. 2. The chemotherapy undertaken by this patient caused acute hemorrhagic cystitis. Which drug was most likely to be responsible for this toxicity? (A) Cyclophosphamide (B) Doxorubicin (C) Fluorouracil (D) Methotrexate (E) Tamoxifen

2. Acrolein, a toxic metabolite of cyclophosphamide that is concentrated in the urine, is associated with hemorrhagic cystitis. Mesna, a sulfur-containing substance that also concentrates in urine, can be administered in an attempt to prevent this complication. The answer is A.

Questions 2-5. A 21-year-old woman from Southeast Asia has been staying with family members in the United States for the last 3 mo and is looking after her sister's preschool children during the day. Because she has difficulty with the English language, her sister escorts her to the emergency department of a local hospital. She tells the staff that her sister has been feeling very tired for the last month, has a poor appetite, and has lost weight. The patient has been feeling somewhat better lately except for a cough that produces a greenish sputum, sometimes specked with blood. With the exception of rales in the left upper lobe, the physical examination is unremarkable and she does not seem to be acutely ill. Laboratory values show a white count of 12,000/μL and a hematocrit of 33%. Chest x-ray film reveals an infiltrate in the left upper lobe with a possible cavity. A Gram-stained smear of the sputum shows mixed flora with no dominance. An acid-fast stain reveals many thin rods of pinkish hue. A preliminary diagnosis is made of pulmonary tuberculosis. Sputum is sent to the laboratory for culture. 2. At this point, the most appropriate course of action is to (A) Hospitalize the patient and start treatment with isoniazid plus rifampin (B) Hospitalize the patient and start treatment with 4 antimycobacterial drugs (C) Prescribe isoniazid for prophylaxis and send the patient home to await culture results (D) Prescribe no drugs and send the patient home to await culture results (E) Treat the patient with isoniazid plus rifampin

2. Despite the fact that this patient does not appear to be acutely ill, she would in most cases be treated with 4 drugs that have activity against M tuberculosis. This is because organisms infecting patients from Southeast Asia are commonly INH-resistant, and coverage must be provided with 3 other antituberculosis drugs in addition to isoniazid. This patient should be hospitalized for several reasons, including potential difficulties with compliance regarding the drug regimen and the fact that young children are in the home where she is living. The answer is B.

2. The mechanism of antibacterial action of doxycycline involves (A) Antagonism of bacterial translocase activity (B) Binding to a component of the 50S ribosomal subunit (C) Inhibition of DNA-dependent RNA polymerase (D) Interference with binding of aminoacyl-tRNA to bacterial ribosomes (E) Selective inhibition of ribosomal peptidyl transferases

2. Doxycycline and other tetracyclines inhibit bacterial protein synthesis by interfering with the binding of aminoacyl-tRNA molecules to bacterial ribosomes. Peptidyl transferase is inhibited by chloramphenicol. The answer is D.

2. A 23-year-old woman desires a combined oral contraceptive for pregnancy protection. Which of the following patient factors would lead a health professional to recommend an alternative form of contraception? (A) Evidence of hirsutism (B) History of gastroesophageal reflux disease and is currently taking omeprazole (C) History of pelvic inflammatory disease (D) History of migraine headache that is well controlled by sumatriptan (E) She plans to use this contraceptive for about 1 yr and will then attempt to become pregnant

2. Estrogen-containing hormonal contraceptives increase the risk of episodes of migraine headache. The answer is D.

Questions 2 and 3. A 30-year-old male patient who is HIVpositive and symptomatic has a CD4 count of 250/μL and a viral RNA load of 15,000 copies/mL. His treatment involves a 3-drug antiviral regimen consisting of zidovudine, didanosine, and ritonavir. In addition, the patient is taking oral valacyclovir for a herpes infection and ketoconazole for oral candidiasis. Because of weight loss, he is taking dronabinol. In addition, verapamil has been prescribed because he suffers from angina. He now complains of anorexia, nausea and vomiting, and abdominal pain. His abdomen is tender in the epigastric area. Laboratory results reveal an amylase activity of 220 units/L, and a preliminary diagnosis is made of acute pancreatitis. 2. If this patient has acute pancreatitis, the drug most likely to be responsible is (A) Didanosine (B) Ketoconazole (C) Ritonavir (D) Valacyclovir (E) Zidovudine

2. Gastrointestinal problems occur with most antiviral drugs used in the HIV-positive patient, and acute pancreatitis has been reported for several reverse transcriptase inhibitors. However, didanosine is the drug most likely to be responsible because its most characteristic adverse effect is a dose-limiting acute pancreatitis. Other risk factors that are relative contraindications to didanosine are advanced AIDS, hypertriglyceridemia, and alcoholism. The answer is A.

2. A 50-kg patient with creatinine clearance of 80 mL/min has a gram-negative infection. Amikacin is administered intramuscularly at a dose of 5 mg/kg every 8 h, and the patient begins to respond. After 2 d, creatinine clearance declines to 40 mL/min. Assuming that no information is available about amikacin plasma levels, what would be the most reasonable approach to management of the patient at this point? (A) Administer 5 mg/kg every 12 h (B) Decrease the dosage to a daily total of 200 mg (C) Decrease the dosage to 125 mg every 8 h (D) Discontinue amikacin and switch to gentamicin (E) Maintain the patient on the present dosage and test auditory function

2. Monitoring plasma drug levels is important when aminoglycosides are used. In this case, the patient seems to be improving, so a decrease of the amikacin dose in proportion to decreased creatinine clearance is most appropriate. Because creatinine clearance is only one half of the starting value, a dose reduction should be made to one half of that given initially. The answer is C.

2. A 34-year-old woman with ulcerative colitis has required long-term treatment with pharmacologic doses of a glucocorticoid agonist. Which of the following is a toxic effect associated with long-term glucocorticoid treatment? (A) A "lupus-like" syndrome (B) Adrenal gland neoplasm (C) Hepatotoxicity (D) Osteoporosis (E) Precocious puberty in children

2. One of the adverse metabolic effects of long-term glucocorticoid therapy is a net loss of bone, which can result in osteoporosis. The answer is D.

2. A 29-year-old woman in her 41st wk of gestation had been in labor for 12 h. Although her uterine contractions had been strong and regular initially, they had diminished in force during the past hour. Which of the following agents would be used to facilitate this woman's labor and delivery? (A) Dopamine (B) Leuprolide (C) Oxytocin (D) Prolactin (E) Vasopressin

2. Oxytocin is an effective stimulant of uterine contraction that is routinely used to augment labor. The answer is C.

2. Plasmodial resistance to chloroquine is due to (A) Change in receptor structure (B) Increase in the activity of DNA repair mechanisms (C) Increased synthesis of dihydrofolate reductase (D) Induction of drug-inactivating enzymes (E) Reduced accumulation of the drug in the food vacuole

2. Resistance to chloroquine in P falciparum can result from decreased accumulation of the drug in the food vacuole caused by the activity of a transporter system encoded by the pfcrt gene. The answer is E.

2. A 24-year-old woman has returned from a vacation abroad suffering from traveler's diarrhea, and her problem has not responded to antidiarrheal drugs. A pathogenic gram-negative bacillus is suspected. Which drug is most likely to be effective in the treatment of this patient? (A) Amoxicillin (B) Ciprofloxacin (C) Sulfacetamide (D) Trimethoprim (E) Vancomycin

2. The second-generation fluoroquinolones are very effective in diarrhea caused by bacterial gram-negative pathogens, including E coli, Shigella, and Salmonella. None of the other drugs listed would be appropriate. Many coliforms are now resistant to amoxicillin and ampicillin. Sulfacetamide is a topical agent used for bacterial conjunctivitis. Although trimethoprim is available as a single drug, resistance emerges rapidly during treatment unless it is used for urinary tract infections, in which high concentrations can be achieved. Vancomycin has no activity against gram-negative bacilli. The answer is B.

2. A nonindigenous person who develops onchocerciasis in an endemic region and receives drug treatment is likely to experience a severe reaction. Symptoms include headache, weakness, rash, muscle aches, hypotension, and peripheral edema. Which statement concerning this reaction is accurate? (A) Bithionol was prescribed (B) Extensive fluid replacement is essential (C) Symptoms are more intense in indigenous adults than expatriate adults (D) The reaction is due to drug toxicity (E) The reaction is due to killing of microfilariae

2. The symptoms described are those of the so-called Mazzotti reaction. They are due to the killing action of ivermectin on microfilariae, and their intensity correlates with skin microfilaria load and is not a drug toxicity. The reaction occurs more frequently and with greater severity in nonindigenous persons than in the indigenous inhabitants of endemic areas. The answer is E.

Questions 1-3. A 24-year-old woman was found to have mild hyperthyroidism due to Graves' disease. She appears to be in good health otherwise. 2. The decision is made to begin treatment with methimazole. Methimazole reduces serum concentration of T3 primarily by which of the following mechanisms? (A) Accelerating the peripheral metabolism of T3 (B) Inhibiting the proteolysis of thyroid-binding globulin (C) Inhibiting the secretion of TSH (D) Inhibiting the uptake of iodide by cells in the thyroid (E) Preventing the addition of iodine to tyrosine residues on thyroglobulin

2. The thioamides (methimazole and propylthiouracil) act in thyroid cells to prevent conversion of tyrosine residues in thyroglobulin to MIT or DIT. The answer is E.

Questions 2 and 3. A 33-year-old man was seen in a clinic with a complaint of dysuria and urethral discharge of yellow pus. He had a painless clean-based ulcer on the penis and nontender enlargement of the regional lymph nodes. Gram stain of the urethral exudate showed gram-negative diplococci within polymorphonucleocytes. The patient informed the clinic staff that he was unemployed and had not eaten a meal for 2 days. 2. The most appropriate treatment of gonorrhea in this patient is (A) Ampicillin orally for 7 d (B) Ceftriaxone intramuscularly as a single dose (C) Procaine penicillin G intramuscularly as a single dose plus oral probenecid (D) Tetracycline orally for 5 d (E) Vancomycin intramuscularly as a single dose

2. The treatments of choice for gonorrhea include a single dose of ceftriaxone (intramuscularly). Because of the high incidence of beta-lactamase-producing gonococci, the use of penicillin G or amoxicillin is no longer appropriate for gonorrhea. Similarly, many strains of gonococci are resistant to tetracyclines. Alternative drugs (not listed) for gonorrhea include cefixime, azithromycin (see Chapter 44) or spectinomycin (see Chapter 45). The answer is B.

Questions 1-3. A 32-year-old woman underwent segmental mastectomy for a breast tumor of 3 cm diameter. Lymph node sampling revealed 2 involved nodes. Because chemotherapy is of established value in her situation, she underwent postoperative treatment with antineoplastic drugs. The regimen consisted of doxorubicin followed by cyclophosphamide/methotrexate/fluorouracil. Adjunctive drugs included tamoxifen because the tumor cells were hormone receptor-positive. 3. After several cycles of chemotherapy, the patient was found to have a high resting pulse rate. A noninvasive radionuclide scan revealed evidence of cardiomyopathy. The drug that is most likely responsible for the cardiac toxicity is (A) Cyclophosphamide (B) Doxorubicin (C) Fluorouracil (D) Methotrexate (E) Tamoxifen

3. A high resting pulse rate is one of the first signs of cardiotoxicity resulting from anthracyclines, which can include arrhythmias, cardiomyopathies, and congestive heart failure. The risk of cardiotoxicity depends on cumulative dosage, so doxorubicin should be discontinued. The answer is B.

3. All of the following statements about the clinical uses of the aminoglycosides are accurate EXCEPT (A) Effective in the treatment of infections caused by Bacteroides fragilis (B) Gentamicin is used with ampicillin for synergistic effects in the treatment of enterococcal endocarditis (C) Netilmicin is more likely to be effective than streptomycin in the treatment of a hospital-acquired infection caused by Serratia marcescens (D) Often used in combination with cephalosporins in the empiric treatment of life-threatening bacterial infections (E) Owing to their polar nature, aminoglycosides are not absorbed after oral administration

3. Aminoglycoside antibiotics act at the ribosomal level and their intracellular accumulation by bacteria is oxygen dependent. Anaerobic bacteria including B fragilis are innately resistant. The answer is A.

3. Clarithromycin and erythromycin have very similar spectra of antimicrobial activity. The major advantage of clarithromycin is that it (A) Does not inhibit hepatic drug-metabolizing enzymes (B) Eradicates mycoplasmal infections in a single dose (C) Has greater activity against M avium-intracellulare complex (D) Is active against methicillin-resistant strains of staphylococci (E) Is active against strains of streptococci that are resistant to erythromycin

3. Clarithromycin can be administered less frequently than erythromycin, but it is not effective in single doses against susceptible organisms. Organisms resistant to erythromycin, including pneumococci and methicillin-resistant staphylococci, are also resistant to other macrolides. Drug interactions have occurred with clarithromycin through its ability to inhibit cytochrome P450. Clarithromycin is more active than erythromycin against M avium complex, T gondii, and H pylori. The answer is C.

3. Men who use large doses of anabolic steroids are at increased risk of which of the following? (A) Anemia (B) Cholestatic jaundice and elevation of aspartate transaminase levels in the blood (C) Hirsutism (D) Hyperprolactinemia (E) Testicular enlargement

3. In men, large doses of anabolic steroids are associated with liver impairment, including cholestasis and elevation of serum concentrations of transaminases. The answer is B.

3. A 46-year-old male patient has Cushing's syndrome that is due to the presence of an adrenal tumor. Which of the following drugs would be expected to reduce the signs and symptoms of this man's disease? (A) Betamethasone (B) Cortisol (C) Fludrocortisone (D) Ketoconazole (E) Triamcinolone

3. Ketoconazole inhibits many types of cytochrome P450 enzymes. It can be used to reduce the unregulated overproduction of corticosteroids by adrenal tumors. The answer is D.

Questions 3-5. A traveler in a geographical region where chloroquine- resistant P falciparum is endemic used a drug for prophylaxis but nevertheless developed a severe attack of P vivax malaria. 3. The drug taken for chemoprophylaxis was probably (A) Atovaquone (B) Diloxanide furoate (C) Mefloquine (D) Proguanil (E) Quinine

3. Mefloquine is a recommended drug for prophylaxis in regions of the world where chloroquine-resistant P falciparum is endemic. One dose of mefloquine weekly starting before travel and continuing until 4 wk after leaving the region is the preferred regimen. Doxycycline is an alternative drug for this indication, as is atovaquone plus proguanil (Malarone). The answer is C.

3. Which statement about pyrantel pamoate is accurate? (A) Acts as an antagonist at GABA receptors (B) Equivalent in efficacy to niclosamide in the treatment of tapeworm infections (C) Hepatotoxicity is dose-limiting (D) Kills adult worms in the colon but not the eggs (E) Synergistic with praziquantel in fluke infections

3. Pyrantel pamoate, an activator of nicotinic receptors, is equivalent to albendazole and mebendazole in the treatment of common nematode infections. It acts on adult worms in the colon, but not on eggs. The drug causes only mild gastrointestinal side effects and is not hepatotoxic. It is not effective in the treatment of infections caused by cestodes or flukes. The answer is D.

Questions 1-3. A 24-year-old woman was found to have mild hyperthyroidism due to Graves' disease. She appears to be in good health otherwise. 3. Though rare, a serious toxicity associated with the thioamides is which of the following? (A) Agranulocytosis (B) Lupus erythematosus-like syndrome (C) Myopathy (D) Torsades de pointes arrhythmia (E) Thrombotic thrombocytic purpura (TTP)

3. Rarely, the thioamides cause severe adverse reactions that include agranulocytosis, vasculitis, hepatic damage, and hypoprothrombinemia. The answer is A.

Questions 2-5. A 21-year-old woman from Southeast Asia has been staying with family members in the United States for the last 3 mo and is looking after her sister's preschool children during the day. Because she has difficulty with the English language, her sister escorts her to the emergency department of a local hospital. She tells the staff that her sister has been feeling very tired for the last month, has a poor appetite, and has lost weight. The patient has been feeling somewhat better lately except for a cough that produces a greenish sputum, sometimes specked with blood. With the exception of rales in the left upper lobe, the physical examination is unremarkable and she does not seem to be acutely ill. Laboratory values show a white count of 12,000/μL and a hematocrit of 33%. Chest x-ray film reveals an infiltrate in the left upper lobe with a possible cavity. A Gram-stained smear of the sputum shows mixed flora with no dominance. An acid-fast stain reveals many thin rods of pinkish hue. A preliminary diagnosis is made of pulmonary tuberculosis. Sputum is sent to the laboratory for culture. 3. Which drug regimen should be initiated in this patient when treatment is started? (A) Amikacin, isoniazid, pyrazinamide, streptomycin (B) Ciprofloxacin, cycloserine, isoniazid, PAS (C) Ethambutol, isoniazid, ofloxacin, streptomycin (D) Ethionamide, pyrazinamide, rifampin, streptomycin (E) Isoniazid, rifampin, pyrazinamide, ethambutol

3. Sputum cultures will not be available for several weeks, and no information is available regarding drug susceptibility of the organism at this stage. For optimum coverage, the initial regimen should include INH, rifampin, pyrazinamide, and ethambutol. INH-resistant organisms are usually sensitive to both rifampin and pyrazinamide. Streptomycin is usually reserved for use in severe forms of tuberculosis or for infections known to be resistant to first-line drugs. Likewise, amikacin and ciprofloxacin are possible agents for treatment of multidrug-resistant strains of M tuberculosis. Cycloserine, PAS, and rifabutin are alternative second-line drugs that may be used in cases of failed response to more conventional agents. The answer is E.

3. Which statement about the clinical use of sulfonamides is false? (A) Active against C trachomatis and can be used topically for the treatment of chlamydial infections of the eye (B) Are not effective as sole agents in the treatment of prostatitis (C) Effective in Rocky Mountain spotted fever in patients allergic to tetracyclines (D) Resistance can occur in some strains of bacteria because of increased production of PABA (E) Some resistant bacterial strains exhibit decreased intracellular accumulation of sulfonamides

3. Sulfonamides have minimal therapeutic actions in rickettsial infections. Chloramphenicol may be used for Rocky Mountain spotted fever in patients with established allergy or other contraindication to tetracyclines. All of the other statements about sulfonamide antimicrobial drugs are accurate. The answer is C.

Questions 2 and 3. A 30-year-old male patient who is HIVpositive and symptomatic has a CD4 count of 250/μL and a viral RNA load of 15,000 copies/mL. His treatment involves a 3-drug antiviral regimen consisting of zidovudine, didanosine, and ritonavir. In addition, the patient is taking oral valacyclovir for a herpes infection and ketoconazole for oral candidiasis. Because of weight loss, he is taking dronabinol. In addition, verapamil has been prescribed because he suffers from angina. He now complains of anorexia, nausea and vomiting, and abdominal pain. His abdomen is tender in the epigastric area. Laboratory results reveal an amylase activity of 220 units/L, and a preliminary diagnosis is made of acute pancreatitis. 3. In the further treatment of this patient, the drug causing the pancreatitis should be withdrawn and replaced by (A) Atazanavir (B) Cidofovir (C) Foscarnet (D) Lamivudine (E) Ribavirin

3. Symptomatic AIDS patients should be treated with a HAART regimen regardless of a relatively high CD4 count or a relatively low HIV RNA load. Because didanosine must be discontinued, lamivudine would be the best choice for replacement in this case. Use of a second protease inhibitor (eg, atazanavir) with a single reverse transcriptase inhibitor could be as effective as regimens that include 2 reverse transcriptase inhibitors, although there may be an increased possibility of drug interactions. Atazanavir use is associated with electrocardiographic PR-interval prolongation, which may be exacerbated by other causative agents such as the calcium channel blocker verapamil, which the patient is taking for angina. The answer is D.

Questions 3-5. A 37-year-old woman with leukemia was undergoing chemotherapy with intravenous antineoplastic drugs. During treatment, she developed a systemic infection from an opportunistic pathogen. There was no erythema or edema at the catheter insertion site. A white vaginal discharge was observed. After appropriate specimens were obtained for culture, empiric antibiotic therapy was started with gentamicin, nafcillin, and ticarcillin intravenously. This regimen was maintained for 72 h, during which time the patient's condition did not improve significantly. Her throat was sore, and white plaques had appeared in her pharynx. On day 4, none of the cultures had shown any bacterial growth, but both the blood and urine cultures grew out Candida albicans. 3. At this point, the best course of action is to (A) Continue current antibiotics and start amphotericin B (B) Continue current antibiotics and start flucytosine (C) Stop current antibiotics and start amphotericin B (D) Stop current antibiotics and start ketoconazole (E) Stop current antibiotics and start terbinafine

3. The antibiotic regimen should be stopped, since the condition of the patient did not improve after 3 d of such treatment, the cultures were negative for bacteria, and the clinical picture suggested that the patient had a fungal infection. This was subsequently confirmed by blood culture. The answer is C.

3. A 3-year-old boy with failure to thrive and metabolic disturbances was found to have an inactivating mutation in the gene that encodes the growth hormone receptor. Which of the following drugs is most likely to improve his metabolic function and promote his growth? (A) Atosiban (B) Bromocriptine (C) Mecasermin (D) Octreotide (E) Somatropin

3. This child's condition is due to the inability of GH to stimulate the production of insulin-like growth factors, the ultimate mediators of GH effects. Mecasermin, a combination of recombinant IGF-1 and the binding protein that protects IGF-1 from immediate destruction, will help correct the IGF deficiency. Because of the inactive GH receptors, somatropin will not be effective. The answer is C.

Questions 2 and 3. A 33-year-old man was seen in a clinic with a complaint of dysuria and urethral discharge of yellow pus. He had a painless clean-based ulcer on the penis and nontender enlargement of the regional lymph nodes. Gram stain of the urethral exudate showed gram-negative diplococci within polymorphonucleocytes. The patient informed the clinic staff that he was unemployed and had not eaten a meal for 2 days. 3. Immunofluorescent microscopic examination of fluid expressed from the penile chancre of this patient revealed treponemes. Because he appears to be infected with Treponema pallidum, the best course of action would be to (A) Administer a single oral dose of fosfomycin (B) Give no other antibiotics because drug treatment of gonorrhea provides coverage for incubating syphilis (C) Inject intramuscular benzathine penicillin G (D) Treat with oral tetracycline for 7 d (E) Treat with vancomycin

3. This patient with gonorrhea also has primary syphilis. The penile chancre, the enlarged nontender lymph nodes, and the microscopic identification of treponemes in fluid expressed from the lesion are essentials of diagnosis. Although a single dose of ceftriaxone may cure incubating syphilis, it cannot be relied on for treating primary syphilis. The most appropriate course of action in this patient is to administer a single intramuscular injection of 2.4 million units of benzathine penicillin G. For penicillin-allergic patients, oral doxycycline or tetracycline for 15 d (not 7 d) is effective in most cases (see Chapter 44). However, lack of compliance may be a problem with oral therapy. Fosfomycin and vancomycin have no significant activity against spirochetes. The answer is C.

4. A newborn girl exhibited ambiguous genitalia, hyponatremia, hyperkalemia, and hypotension as a result of genetic deficiency of 21β-hydroxylase activity. Treatment consisted of fluid and salt replacement and hydrocortisone administration. In this type of adrenal hyperplasia in which there is excess production of cortisol precursors, which of the following describes the primary therapeutic effect of glucocorticoid administration? (A) Increased adrenal estrogen synthesis (B) Inhibition of adrenal aldosterone synthesis (C) Prevention of hypoglycemia (D) Recovery of normal immune function (E) Suppression of ACTH secretion

4. A 21β-hydroxylase deficiency prevents normal synthesis of cortisol and aldosterone, and causes accumulation of cortisol precursors (Figure 39-2). The hypothalamic-pituitary system responds to the abnormally low levels of cortisol by increasing ACTH release. High levels of ACTH induce adrenal hyperplasia and excess production of adrenal androgens, which can cause virilization of females and prepubertal males. Glucocorticoid is administered to replace the missing mineralocorticoid and glucocorticoid activity and to suppress ACTH release, which removes the stimulus for excess adrenal androgen production. The answer is E.

4. A 31-year-old man has gonorrhea. He has no drug allergies, but a few years ago acute hemolysis followed use of an antimalarial drug. The physician is concerned that the patient has an accompanying urethritis caused by C trachomatis, although no cultures or enzyme tests have been performed. Which of the following drugs will be reliably effective against both gonococci and C trachomatis and safe to use in this patient? (A) Cefixime (B) Ciprofloxacin (C) Spectinomycin (D) Sulfamethoxazole-trimethoprim (E) None of the above

4. Although cefixime in a single oral dose is effective in gonorrhea (Chapter 43), it has no activity against organisms causing nongonococcal urethritis. Spectinomycin (Chapter 45) is active against most gonococci, but does not eradicate a urogenital chlamydial infection. Although ciprofloxacin might be effective in both gonorrhea and chlamydial urethritis, it is no longer recommended for treatment of gonorrhea in the United States, since resistance is now common. This patient could be treated by single oral doses of cefixime plus azithromycin (not listed). Sulfamethoxazole or TMP-SMZ would not be useful and may cause acute hemolysis in this patient. The answer is E.

Questions 3-5. A traveler in a geographical region where chloroquine- resistant P falciparum is endemic used a drug for prophylaxis but nevertheless developed a severe attack of P vivax malaria. 4. Which drug should be used for oral treatment of the acute attack of P vivax malaria but does not eradicate exoerythrocytic forms of the parasite? (A) Chloroquine (B) Mefloquine (C) Primaquine (D) Pyrimethamine-sulfadoxine (E) Quinidine

4. Chloroquine is the drug of choice for the oral treatment of an acute attack of malaria caused by P vivax but will not eradicate exoerythrocytic forms of the parasite. The answer is A.

4. A 56-year-old woman presented to the emergency department with tachycardia, shortness of breath, and chest pain. She had had shortness of breath and diarrhea for the last 2 d and was sweating and anxious. A relative reported that the patient had run out of methimazole 2 wk earlier. A TSH measurement revealed a value of <0.01 mIU/L (normal 0.4-4.0 mIU/L). The diagnosis of thyroid storm was made. Which of the following is a drug that is a useful adjuvant in the treatment of thyroid storm? (A) Amiodarone (B) Betamethasone (C) Epinephrine (D) Propranolol (E) Radioactive iodine

4. In thyroid storm, β blockers such as propranolol are useful in controlling the tachycardia and other cardiac abnormalities, and propranolol also inhibits peripheral conversion of T4 to T3. The answer is D.

Questions 3-5. A 37-year-old woman with leukemia was undergoing chemotherapy with intravenous antineoplastic drugs. During treatment, she developed a systemic infection from an opportunistic pathogen. There was no erythema or edema at the catheter insertion site. A white vaginal discharge was observed. After appropriate specimens were obtained for culture, empiric antibiotic therapy was started with gentamicin, nafcillin, and ticarcillin intravenously. This regimen was maintained for 72 h, during which time the patient's condition did not improve significantly. Her throat was sore, and white plaques had appeared in her pharynx. On day 4, none of the cultures had shown any bacterial growth, but both the blood and urine cultures grew out Candida albicans. 4. If amphotericin B is administered, the patient should be premedicated with (A) Diphenhydramine (B) Ibuprofen (C) Prednisone (D) Any or all of the above (E) None of the above

4. Infusion-related adverse effects of amphotericin B include chills and fevers (the "shake and bake" syndrome), muscle spasms, nausea, headache, and hypotension. Antipyretics, antihistamines, and glucocorticoids all have been shown to be helpful. The administration of a 1-mg test dose of amphotericin B is sometimes useful in predicting the severity of infusion-related toxicity. The answer is D.

4. An important difference between leuprolide and ganirelix is that ganirelix (A) Can be administered as an oral formulation (B) Can be used alone to restore fertility to hypogonadal men and women (C) Immediately reduces gonadotropin secretion (D) Initially stimulates pituitary production of LH and FSH (E) Must be administered in a pulsatile fashion

4. Leuprolide is an agonist of GnRH receptors, whereas ganirelix is an antagonist. Although both drugs can be used to inhibit gonadotropin release, ganirelix does so immediately, whereas leuprolide does so only after about 1 wk of sustained activity. The answer is C.

4. Which statement about imipenem is accurate? (A) Active against methicillin-resistant staphylococci (B) Has a narrow spectrum of antibacterial action (C) In renal dysfunction, dosage reduction is necessary to avoid seizures (D) Is highly susceptible to beta-lactamases produced by Enterobacter species (E) Is used in fixed combination with sulbactam

4. Like other carbapenems, imipenem has a wide spectrum of activity that includes anaerobes and many beta-lactamaseproducing gram-negative rods, including Enterobacter. However, the carbapenems are not active against MRSA strains. Imipenem is rapidly hydrolyzed by renal dehydropeptidases and is given in combination with cilastatin, an inhibitor of this enzyme. Severe CNS toxicity, including seizures, will occur if the dose of imipenem is not reduced in patients with renal impairment. The answer is C.

4. The primary mechanism of resistance of gram-positive organisms to erythromycin is (A) Decreased activity of uptake mechanisms (B) Decreased drug permeability of the cytoplasmic membrane (C) Formation of drug-inactivating acetyltransferases (D) Formation of esterases that hydrolyze the lactone ring (E) Methylation of binding sites on the 50S ribosomal subunit

4. Methylase production and methylation of the receptor site are established mechanisms of resistance of gram-positive organisms to macrolide antibiotics. Such enzymes may be inducible by macrolides or constitutive; in the latter case, cross-resistance occurs between macrolides and clindamycin. Increased expression of efflux pumps is also a mechanism of macrolide resistance. Esterase formation is a mechanism of macrolide resistance seen in coliforms. The answer is E.

4. A student studying medicine at a Caribbean university develops fever, chills, and diarrhea resulting from S mansoni, and oxamniquine is prescribed. Which statement about the proposed drug therapy is accurate? (A) If the patient has a history of seizure disorders, hospitalization is recommended during treatment (B) It is not effective in the late stages of the disease (C) Oxamniquine is safe to use in pregnancy (D) The drug blocks GABA receptors in trematodes (E) The drug is also effective in tapeworm infections

4. Oxamniquine may cause seizures, especially in persons with a history of convulsive disorders. Such persons should be hospitalized or treated with praziquantel. Oxamniquine is effective in all stages of disease caused by S mansoni, including advanced hepatosplenomegaly, and it has been used extensively for mass treatment. The drug is not effective in other schistosomal diseases, and it is contraindicated in pregnancy. The answer is A.

Questions 2-5. A 21-year-old woman from Southeast Asia has been staying with family members in the United States for the last 3 mo and is looking after her sister's preschool children during the day. Because she has difficulty with the English language, her sister escorts her to the emergency department of a local hospital. She tells the staff that her sister has been feeling very tired for the last month, has a poor appetite, and has lost weight. The patient has been feeling somewhat better lately except for a cough that produces a greenish sputum, sometimes specked with blood. With the exception of rales in the left upper lobe, the physical examination is unremarkable and she does not seem to be acutely ill. Laboratory values show a white count of 12,000/μL and a hematocrit of 33%. Chest x-ray film reveals an infiltrate in the left upper lobe with a possible cavity. A Gram-stained smear of the sputum shows mixed flora with no dominance. An acid-fast stain reveals many thin rods of pinkish hue. A preliminary diagnosis is made of pulmonary tuberculosis. Sputum is sent to the laboratory for culture. 4. Which statement concerning the possible use of isoniazid (INH) in this patient is false? (A) A lower maintenance dose than usual is required in a patient from Southeast Asia (B) Flushing, sweating, dyspnea, and palpitations may occur after ingestion of tyramine-containing foods (C) Peripheral neuritis may occur during treatment (D) The patient should take pyridoxine daily (E) The risk of the patient developing hepatitis from INH is less than 2%

4. Patients from Pacific Rim countries do not require lower doses of INH. Fast acetylators, including Native Americans, may require higher doses of the drug than others. Peripheral neuropathy caused by INH is due to pyridoxine deficiency. It is more common in the diabetic, malnourished, or AIDS patient and can be prevented by a daily dose of 25-50 mg of pyridoxine. INH can inhibit monoamine oxidase type A and has caused tyramine reactions. Hepatotoxicity is age-dependent, with an incidence of 0.3% in patients aged 21-35 yr and greater than 2% in patients older than 50 yr. The answer is A.

4. A 50-year-old woman with a positive mammogram undergoes lumpectomy and a small carcinoma is removed. Biochemical analysis of the cancer reveals the presence of estrogen and progesterone receptors. After this procedure, she will probably receive which of the following drugs? (A) Danazol (B) Flutamide (C) Leuprolide (D) Mifepristone (E) Tamoxifen

4. Tamoxifen has proved useful in adjunctive therapy of breast cancer; the drug decreases the rate of recurrence of cancer. The answer is E.

4. Which statement is accurate regarding the antibacterial action of gentamicin? (A) Antibacterial activity is often reduced by the presence of an inhibitor of cell wall synthesis (B) Antibacterial action is not concentration-dependent (C) Antibacterial action is time-dependent (D) Efficacy is directly proportional to the duration of time that the plasma level is greater than the minimal inhibitory concentration (E) Gentamicin continues to exert antibacterial effects even after plasma levels decrease below detectable levels

4. The antibacterial action of aminoglycosides is concentration dependent rather than time dependent. The activity of gentamicin continues to increase as its plasma level rises above the minimal inhibitory concentration (MIC). When the plasma level falls below the MIC, gentamicin continues to exert antibacterial effects for several hours, exerting a post-antibiotic effect. Inhibitors of bacterial cell wall synthesis often exert synergistic effects with aminoglycosides, possibly by increasing the intracellular accumulation of the aminoglycoside. The answer is E.

4. Which of the following is a cell cycle-specific anticancer drug that acts mainly in the M phase of the cell cycle? (A) Bleomycin (B) Cisplatin (C) Etoposide (D) Methotrexate (E) Paclitaxel

4. The taxanes, paclitaxel and docetaxel, interfere with the separation of chromosomes during mitosis because of their effects on microtubules. The answer is E.

4. In an accidental needlestick, an unknown quantity of blood from an AIDS patient is injected into a resident physician. The most recent laboratory report on the AIDS patient shows a CD4 count of 20/μL and a viral RNA load of greater than 107 copies/mL. The most appropriate course of action regarding treatment of the resident is to (A) Monitor the resident's blood to determine whether HIV transmission has occurred (B) Treat with single doses of ritonavir and zidovudine (C) Treat with full doses of zidovudine for 2 wk (D) Treat with full doses of zidovudine for 4 wk (E) Treat with zidovudine plus lamivudine plus a protease inhibitor for 4 wk

4. The viral RNA titer in the blood from the AIDS patient in this case is very high, and this needlestick must be considered as a high-risk situation. Although full doses of zidovudine for 4 wk has been shown to have prophylactic value, in highrisk situations combination regimens are favored. Optimal prophylaxis in this case might best be provided by the combination of zidovudine with lamivudine (basic regimen), plus the addition of protease inhibitors (expanded regimen). The answer is E.

5. Which of the following best describes a glucocorticoid response element? (A) A protein regulator that controls the interaction between an activated steroid receptor and DNA (B) A short DNA sequence that binds tightly to RNA polymerase (C) A small protein that binds to an unoccupied steroid receptor protein and prevents it from becoming denatured (D) A specific nucleotide sequence that is recognized by a steroid hormone receptor-hormone complex (E) The portion of the steroid receptor that binds to DNA

5. Activated steroid hormone receptors mediate their effects on gene expression by binding to hormone response elements, which are short sequences of DNA located near steroidregulated genes. The answer is D.

5. Which statement about the fluoroquinolones is accurate? (A) A fluoroquinolone is the drug of choice for treatment of an uncomplicated urinary tract infection in a 7-year-old girl (B) Antacids increase the oral bioavailability of fluoroquinolones (C) Gonococcal resistance to fluoroquinolones may involve changes in DNA gyrase (D) Modification of moxifloxacin dosage is required in patients when creatinine clearance is less than 50 mL/min (E) The fluoroquinolones are contraindicated in patients with hepatic dysfunction

5. Antacids can decrease oral bioavailability of fluoroquinolones. Neither hepatic or renal dysfunction is a contraindication to the use of fluoroquinolones. Most fluoroquinolones undergo renal elimination and dosage should be modified with creatinine clearance < 50 mL/min. Moxifloxacin elimination occurs mainly via the liver. The fluoroquinolones should not be used to treat uncomplicated first-time urinary tract infections in children because of possible effects on cartilage development. Uncomplicated urinary tract infections in children are usually due to a strain of E coli that is sensitive to many other drugs, including beta-lactam antibiotics. The answer is C.

5. A 60-year-old man is found to have a prostate lump and an elevated prostate-specific antigen (PSA) blood test. Magnetic resonance imaging suggests several enlarged lymph nodes in the lower abdomen, and an x-ray reveals 2 radiolucent lesions in the bony pelvis. This patient is likely to be treated with which of the following drugs? (A) Anastrozole (B) Desogestrel (C) Flutamide (D) Methyltestosterone (E) Oxandrolone

5. Antiandrogen drugs are used to treat metastatic prostate cancer because they have efficacy, whereas conventional cytotoxic drugs do not. Flutamide is a competitive antagonist of the androgen receptor that is used in combination with a GnRH agonist in the treatment of men with prostate cancer. The answer is C.

5. A 27-year-old woman with amenorrhea, infertility, and galactorrhea was treated with a drug that successfully restored ovulation and menstruation. Before being given the drug, the woman was carefully questioned about previous mental health problems, which she did not have. She was advised to take the drug orally. Which of the following is most likely to be the drug that was used to treat this patient? (A) Bromocriptine (B) Desmopressin (C) Human gonadotropin hormone (D) Leuprolide (E) Octreotide

5. Bromocriptine, a dopamine receptor agonist, is used to treat the amenorrhea-galactorrhea syndrome, which is a consequence of hyperprolactinemia. Because of its central dopaminergic effects, the drug should not be used in patients with a history of schizophrenia or other forms of psychotic illness. The answer is A.

5. A 26-year-old woman was treated for a suspected chlamydial infection at a neighborhood clinic. She was given a prescription for oral doxycycline to be taken for 10 d. Three weeks later, she returned to the clinic with a mucopurulent cervicitis. On questioning she admitted not having the prescription filled. The best course of action at this point would be to (A) Delay drug treatment until the infecting organism is identified (B) Rewrite the original prescription for oral doxycycline (C) Treat her in the clinic with a single oral dose of cefixime (D) Treat her in the clinic with a single oral dose of azithromycin (E) Write a prescription for oral erythromycin for 10 d

5. Cervicitis or urethritis is often caused by C trachomatis. Such infections may develop slowly because of the long incubation period of chlamydial infection. Treatment with oral doxycycline for 14 d (as originally prescribed) would have eradicated C trachomatis and most other organisms commonly associated with nongonococcal cervicitis or urethritis. Given the limited compliance of this patient, the best course of action would be the administration (in the clinic) of a single oral dose of azithromycin. The answer is D.

5. A 36-year-old woman recently treated for leukemia is admitted to the hospital with malaise, chills, and high fever. Gram stain of blood reveals the presence of gram-negative bacilli. The initial diagnosis is bacteremia, and parenteral antibiotics are indicated. The records of the patient reveal that she had a severe urticarial rash, hypotension, and respiratory difficulty after oral penicillin V about 6 mo ago. The most appropriate drug regimen for empiric treatment is (A) Aztreonam (B) Cefazolin (C) Imipenem (D) Nafcillin (E) Ticarcillin plus clavulanic acid

5. Each of the drugs listed has activity against some gramnegative bacilli. All penicillins should be avoided in patients with a history of allergic reactions to any individual penicillin drug. Cephalosporins should also be avoided in patients who have had anaphylaxis or other severe hypersensitivity reactions after use of a penicillin. There is partial cross-reactivity between penicillins and the carbapenems such as imipenem and meropenem, but no cross-reactivity between the penicillins and aztreonam. The answer is A.

Questions 5 and 6. A patient with AIDS has a CD4 count of 45/μL. He is being maintained on a 3-drug regimen of indinavir, didanosine, and zidovudine. For prophylaxis against opportunistic infections, he is also receiving ganciclovir, fluconazole, rifabutin, and trimethoprim-sulfamethoxazole. 5. The drug most likely to suppress herpetic infections and provide prophylaxis against CMV retinitis in this patient is (A) Fluconazole (B) Ganciclovir (C) Indinavir (D) Rifabutin (E) Trimethoprim-sulfamethoxazole

5. Ganciclovir has been the most commonly used drug for prevention and treatment of CMV infections in the immunocompromised patient. Cidofovir (not listed) is also very effective in CMV retinitis and has good activity against many strains of HSV, including those resistant to acyclovir. The answer is B.

Questions 3-5. A 37-year-old woman with leukemia was undergoing chemotherapy with intravenous antineoplastic drugs. During treatment, she developed a systemic infection from an opportunistic pathogen. There was no erythema or edema at the catheter insertion site. A white vaginal discharge was observed. After appropriate specimens were obtained for culture, empiric antibiotic therapy was started with gentamicin, nafcillin, and ticarcillin intravenously. This regimen was maintained for 72 h, during which time the patient's condition did not improve significantly. Her throat was sore, and white plaques had appeared in her pharynx. On day 4, none of the cultures had shown any bacterial growth, but both the blood and urine cultures grew out Candida albicans. 5. Candida is a major cause of nosocomial bloodstream infection. The opportunistic fungal infection in this patient could have been prevented by administration of (A) Caspofungin (B) Fluconazole (C) Nystatin (D) Posaconazole (E) None of the above

5. In the case of opportunistic candidal infections in the immunocompromised patient, no prophylactic drugs have been shown to be clinically effective. Prophylaxis against other fungi may be effective in some instances, including suppression of cryptococcal meningitis in AIDS patients with fluconazole. However, prophylactic use of azoles may contribute to the development of fungal resistance. The answer is E.

5. A 65-year-old man with multinodular goiter is scheduled for a near-total thyroidectomy. Which of the following drugs will be administered for 10-14 d before surgery to reduce the vascularity of his thyroid gland? (A) Levothyroxine (B) Liothyronine (C) Lugol's solution (D) Prednisone (E) Radioactive iodine

5. Iodides inhibit the synthesis and release of thyroid hormone and decrease the size and vascularity of the hyperplastic gland. Lugol's solution contains a mixture of potassium iodide and iodine. The answer is C.

5. An adult patient is being treated for acute leukemia with a combination of anticancer drugs that includes cyclophosphamide, mercaptopurine, methotrexate, vincristine, and prednisone. He is also using ondansetron for emesis, a chlorhexidine mouthwash to reduce mucositis, and laxatives. The patient complains of "pins and needle" sensations in the extremities and muscle weakness. He is not able to execute a deep knee bend or get up out of a chair without using his arm muscles. He is also very constipated. If these problems are related to the chemotherapy, which of the following is the most likely causative agent? (A) Cyclophosphamide (B) Mercaptopurine (C) Methotrexate (D) Prednisone (E) Vincristine

5. Neuropathy is a toxic side effect of vincristine. In its mildest form, paresthesias occur, but it progresses to significant muscle weakness, initially in the quadriceps muscle group. Constipation is the most common symptom of autonomic neuropathy. The answer is E.

5. A 22-year-old South Korean man has recently moved to Minnesota. He has symptoms of clonorchiasis (anorexia, upper abdominal pain, eosinophilia), presumably contracted in his homeland where the Oriental liver fluke is endemic. He also has symptoms of diphyllobothriasis (abdominal discomfort, diarrhea, megaloblastic anemia), probably caused by consumption of raw fish from lakes near the Canadian border. Which drug is most likely to be effective in the treatment of both clonorchiasis and diphyllobothriasis in this patient? (A) Ivermectin (B) Niclosamide (C) Praziquantel (D) Thiabendazole (E) Pyrantel pamoate

5. Praziquantel is a primary drug for treatment of infections caused by the Oriental liver fluke and by the fish tapeworm. Both types of infection are transmitted mainly via the consumption of raw fish. Niclosamide is also a primary drug for fish tapeworm infections, but it is not active against Clonorchis sinensis. Albendazole is not effective in fish tapeworm infections, but it is useful in the pork tapeworm larval stage (cysticercosis). Pyrantel pamoate is not active against cestodes or trematodes. The answer is C.

Questions 3-5. A traveler in a geographical region where chloroquine- resistant P falciparum is endemic used a drug for prophylaxis but nevertheless developed a severe attack of P vivax malaria. 5. Which drug should be given later to eradicate schizonts and latent hypnozoites in the patient's liver? (A) Amodiaquine (B) Halofantrine (C) Primaquine (D) Quinine (E) Sulfadoxine

5. Primaquine is the only antimalarial drug that reliably acts on tissue schizonts in liver cells. Quinine is a highly effective blood shizonticide against all 4 species of human malaria parasites, but it is not active against liver stages. Starting about day 4 after an acute attack, primaquine should be given daily for 2 wk. The answer is C.

Questions 2-5. A 21-year-old woman from Southeast Asia has been staying with family members in the United States for the last 3 mo and is looking after her sister's preschool children during the day. Because she has difficulty with the English language, her sister escorts her to the emergency department of a local hospital. She tells the staff that her sister has been feeling very tired for the last month, has a poor appetite, and has lost weight. The patient has been feeling somewhat better lately except for a cough that produces a greenish sputum, sometimes specked with blood. With the exception of rales in the left upper lobe, the physical examination is unremarkable and she does not seem to be acutely ill. Laboratory values show a white count of 12,000/μL and a hematocrit of 33%. Chest x-ray film reveals an infiltrate in the left upper lobe with a possible cavity. A Gram-stained smear of the sputum shows mixed flora with no dominance. An acid-fast stain reveals many thin rods of pinkish hue. A preliminary diagnosis is made of pulmonary tuberculosis. Sputum is sent to the laboratory for culture. 5. On her release from the hospital, the patient is advised not to rely solely on oral contraceptives to prevent pregnancy because they may be less effective while she is being maintained on antimycobacterial drugs. The agent most likely to interfere with the action of oral contraceptives is (A) Amikacin (B) Ethambutol (C) Isoniazid (D) Pyrazinamide (E) Rifampin

5. Rifampin induces the formation of several microsomal drugmetabolizing enzymes, including cytochrome P450 isoforms. This action increases the rate of elimination of a number of drugs, including anticoagulants, ketoconazole, methadone, and steroids that are present in oral contraceptives. The pharmacologic activity of these drugs can be reduced markedly in patients taking rifampin. The answer is E.

5. An adult patient (weight 60 kg) has bacteremia suspected to be due to a gram-negative rod. Tobramycin is to be administered using a once-daily dosing regimen, and the loading dose must be calculated to achieve a peak plasma level of 20 mg/L. Assume that the patient has normal renal function. Pharmacokinetic parameters of tobramycin in this patient are as follows: Vd = 20 L; t1/2 = 3 h; CL = 80 mL/min. What loading dose should be given? (A) 100 mg (B) 200 mg (C) 400 mg (D) 600 mg (E) 800 mg

5. The loading dose of any drug is calculated by multiplying the desired plasma concentration (mg/L) by the volume of distribution (L). The answer is C.

Questions 6 and 7. A 28-year-old man living on the East Coast was transferred by his employer to central California for several months. On his return, he complains of having influenza-like symptoms with fever and a cough. He also has red, tender nodules on his shins. His physician suspects that these symptoms are due to coccidioidomycosis contracted during his stay in California. 6. This patient should be treated immediately with (A) Amphotericin B (B) Caspofungin (C) Terbinafine (D) Voriconazole (E) None of these drugs

6. A travel history can be important in the diagnosis of fungal disease. If this patient has a fungal infection of the lungs, it is probably due to C immitis, which is endemic in dry regions of the western United States. Pulmonary symptoms of coccidioidomycosis are usually self-limiting, and drug therapy is not commonly required in an otherwise healthy patient. Tender red nodules on extensor surfaces constitute a good prognostic sign. Erythema nodosum is a delayed hypersensitivity response to fungal antigens. No organisms are present in the lesions, and it is not a sign of disseminated disease. The answer is E.

6. A patient with AIDS and a CD4 cell count of 100/μL has persistent fever and weight loss associated with invasive pulmonary disease due to M avium complex (MAC). Optimal management of this patient is to (A) Select an antibiotic based on drug susceptibility of the cultured organism (B) Start treatment with INH and pyrazinamide (C) Treat with rifabutin because it prevents the development of MAC bacteremia (D) Treat with the combination of clarithromycin, ethambutol, and rifabutin (E) Treat with trimethoprim-sulfamethoxazole

6. Combinations of antibiotics are essential for suppression of disease caused by M avium complex in the AIDS patient, and treatment should be started before culture results are available. Although rifabutin is prophylactic against MAC bacteremia when it is used as sole therapy in active disease, resistant strains of the organism emerge rapidly. MAC is much less susceptible than M tuberculosis to conventional antimycobacterial drugs. Currently, the optimum regimen consists of clarithromycin (or azithromycin) with ethambutol and rifabutin. The answer is D.

Questions 5 and 6. A patient with AIDS has a CD4 count of 45/μL. He is being maintained on a 3-drug regimen of indinavir, didanosine, and zidovudine. For prophylaxis against opportunistic infections, he is also receiving ganciclovir, fluconazole, rifabutin, and trimethoprim-sulfamethoxazole. 6. The dose of indinavir in this patient may need to be increased above normal. This is because (A) Fluconazole slows gastric emptying (B) Ganciclovir increases the renal clearance of indinavir (C) Indinavir has to be taken with meals (D) Rifabutin increases liver drug-metabolizing enzymes (E) Sulfamethoxazole displaces indinavir from plasma proteins

6. Drug interactions can be severe in the immunocompromised patient because many of the drugs administered can influence the pharmacokinetic properties of other drugs. Rifabutin, like rifampin, acts as an inducer of several isoforms of hepatic cytochrome P450. This action can result in an increased clearance of other drugs, including indinavir. The answer is D.

6. Glucocorticoids have proved useful in the treatment of which of the following medical conditions? (A) Chemotherapy-induced vomiting (B) Essential hypertension (C) Hyperprolactinemia (D) Parkinson's disease (E) Type II diabetes

6. Glucocorticoids are used in combination with other antiemetics to prevent chemotherapy-induced nausea and vomiting, which are commonly associated with anticancer drugs. The answer is A.

6. A young woman complains of abdominal pain at the time of menstruation. Careful evaluation indicates the presence of significant endometrial deposits on the pelvic peritoneum. Which of the following is the most appropriate medical therapy for this patient? (A) Flutamide, orally (B) Medroxyprogesterone acetate by intramuscular injection (C) Norgestrel as an IUD (D) Oxandrolone by intramuscular injection (E) Raloxifene orally

6. In endometriosis, suppression of ovarian function and production of gonadal steroids are useful. Intramuscular injection of relatively large doses of medroxyprogesterone provides 3 mo of an ovarian suppressive effect because of inhibition of pituitary production of gonadotropins. The answer is B.

6. Which helminthic infection does not respond to treatment with praziquantel? (A) Hydatid disease (B) Opisthorchiasis (C) Paragonimiasis (D) Pork tapeworm infection (E) Schistosomiasis

6. In hydatid disease, praziquantel has marginal efficacy because it does not affect the inner germinal membrane of Echinococcus granulosus present in hydatid cysts. The answer is A.

6. Which of the following is a sign or symptom that would be expected to occur in the event of chronic overdose with exogenous T4? (A) Bradycardia (B) Dry, puffy skin (C) Large tongue and drooping of the eyelids (D) Lethargy, sleepiness (E) Weight loss

6. In hyperthyroidism, the metabolic rate increases, and even though there is increased appetite, weight loss often occurs. The other choices are symptoms seen in hypothyroidism. The answer is E.

6. Who is least likely to be treated with somatropin? (A) A 3-year-old cow on a dairy farm (B) A 4-year-old girl with an XO genetic genotype (C) A 4-year-old boy with chronic renal failure and growth deficiency (D) A 10-year-old boy with polydipsia and polyuria (E) A 37-year-old patient with AIDS-related wasting syndrome

6. Somatropin, recombinant human GH, promotes growth in children with Turner's syndrome (an XO genetic genotype) or chronic renal failure. It also helps combat the AIDSassociated wasting syndrome. Bovine GH promotes milk production in cows. GH would not be appropriate for the boy with polydipsia and polyuria, which is probably symptomatic of a form of diabetes. The answer is D.

6. A 55-year-old man complains of periodic bouts of diarrhea with lower abdominal cramping and intermittent rectal bleeding. Seen in the clinic, he appears well nourished, with blood pressure in the normal range. Examination reveals moderate abdominal pain and tenderness. His current medications are limited to loperamide for his diarrhea. Sigmoidoscopy reveals mucosal edema, friability, and some pus. Laboratory findings include mild anemia and decreased serum albumin. Microbiologic examination via stool cultures and mucosal biopsies do not reveal any evidence for bacterial, amebic, or cytomegalovirus involvement. The most appropriate drug to use in this patient is (A) Amoxicillin (B) Ciprofloxacin (C) Doxycycline (D) Sulfasalazine (E) Trimethoprim-sulfamethoxazole

6. In the absence of any evidence pointing toward a definite microbial cause for the colitis in this patient, a drug that decreases inflammation is indicated. Sulfasalazine has significant anti-inflammatory action, and its oral use results in symptomatic improvement in 50-75% of patients suffering from ulcerative colitis. The drug is also used for its anti-inflammatory effects in rheumatoid arthritis. The answer is D.

Questions 6 and 7. A male patient presents with lower abdominal discomfort, flatulence, and occasional diarrhea. A diagnosis of intestinal amebiasis is made, and E histolytica is identified in his diarrheal stools. An oral drug is prescribed, which reduces his intestinal symptoms. Later he presents with severe dysentery, right upper quadrant pain, weight loss, fever, and an enlarged liver. Amebic liver abscess is diagnosed, and the patient is hospitalized. He has a recent history of drug treatment for a tachyarrhythmia. 6. The preferred treatment that he should have received for the initial symptoms (which were indicative of mild to moderate disease) is (A) Diloxanide furoate (B) Iodoquinol (C) Metronidazole (D) Metronidazole plus diloxanide furoate (E) Paromomycin

6. Metronidazole plus a luminal amebicide is the treatment of choice in mild to moderate amebic colitis. Diloxanide furoate (or iodoquinol, or paramomycin) can be used as the sole agent in asymptomatic intestinal infection. The answer is D.

6. Which of the following is a drug that is used in combination therapy for testicular carcinoma and is also associated with nephrotoxicity? (A) Bleomycin (B) Cisplatin (C) Etoposide (D) Leuprolide (E) Vinblastine

6. Nephrotoxicity is a characteristic toxicity of cisplatin. Renal toxicity can be reduced by slow intravenous infusion, maintenance of good hydration, and administration of mannitol to maximize urine flow. For testicular cancer, cisplatin is used in combination with etoposide and bleomycin. The answer is B.

Questions 6-8. A 52-year-old man (weight 70 kg) is brought to the hospital emergency department in a confused and delirious state. He has had an elevated temperature for more than 24 h, during which time he had complained of a severe headache and had suffered from nausea and vomiting. Lumbar puncture reveals an elevated opening pressure, and cerebrospinal fluid findings include elevated protein, decreased glucose, and increased neutrophils. Gram stain of a smear of cerebrospinal fluid reveals gram-positive diplococci, and a preliminary diagnosis is made of purulent meningitis. The microbiology report informs you that for approximately 15% of S pneumoniae isolates in the community, the minimal inhibitory concentration for penicillin G is 20 mcg/mL. 6. Treatment of this patient should be initiated immediately with intravenous administration of (A) Ampicillin-sulbactam (B) Cefazolin (C) Cefotaxime plus vancomycin (D) Nafcillin (E) Ticarcillin

6. Pneumococcal isolates with a minimal inhibitory concentration for penicillin G of greater than 2 mcg/mL are highly resistant. Such strains are not killed by the concentrations of penicillin G or ampicillin that can be achieved in the cerebrospinal fluid. Nafcillin has minimal activity against penicillin-resistant pneumococci, and ticarcillin is used mainly for infections caused by gram-negative rods. Cefotaxime and ceftriaxone are the most active cephalosporins against penicillin-resistant pneumococci, and the addition of vancomycin is recommended in the case of highly resistant strains. The answer is C.

6. A 67-year-old man is seen in a hospital emergency department complaining of pain in and behind the right ear. Physical examination shows edema of the external otic canal with purulent exudate and weakness of the muscles on the right side of the face. The patient informs the physician that he is a diabetic. Gram stain of the exudate from the ear shows many polymorphonucleocytes and gram-negative rods, and samples are sent to the microbiology laboratory for culture and drug susceptibility testing. A preliminary diagnosis is made of external otitis. At this point, which of the following is most appropriate? (A) Amikacin should be administered by intramuscular injection, and the patient should be sent home (B) Analgesics should be prescribed for pain, but antibiotics should be withheld pending the results of cultures (C) Oral cefaclor should be prescribed together with analgesics, and the patient should be sent home (D) The patient should be hospitalized and treatment started with gentamicin plus ticarcillin (E) The patient should be hospitalized and treatment started with intravenous imipenem-cilastatin

6. The diabetic patient with external otitis is at special risk because of the danger of spread to the middle ear and possibly the meninges, so hospitalization is advisable, especially in the elderly. Likely pathogens include E coli and Pseudomonas aeruginosa, and coverage must be provided for these and possibly other gram-negative rods. The combination of an aminoglycoside plus a wider spectrum penicillin is most suitable in this case and is synergistic against many pseudomonas strains. Imipenem-cilastatin is also possible, but resistant strains of P aeruginosa have emerged during treatment. Cefaclor lacks antipseudomonal activity. The answer is D.

6. A 55-year-old patient with a prosthetic heart valve is to undergo a periodontal procedure involving scaling and root planing. Several years ago, the patient had a severe allergic reaction to procaine penicillin G. Regarding prophylaxis against bacterial endocarditis, which one of the following drugs taken orally is most appropriate? (A) Amoxicillin 10 min before the procedure (B) Clindamycin 1 h before the procedure (C) Erythromycin 1 h before the procedure and 4 h after the procedure (D) Vancomycin 15 min before the procedure (E) No prophylaxis is needed because this patient is in the negligible risk category

6. This patient is in the high-risk category for bacterial endocarditis and should receive prophylactic antibiotics before many dental procedures. The American Heart Association recommends that clindamycin be used in patients allergic to penicillins. Oral erythromycin is not recommended because it is no more effective than clindamycin and causes more gastrointestinal side effects. Intravenous vancomycin (not oral), sometimes with gentamicin, is recommended for prophylaxis in high-risk penicillin-allergic patients undergoing genitourinary and lower gastrointestinal surgical procedures. Complete cross-allergenicity must be assumed between individual penicillins. The answer is B.

7. A 10-year-old boy has uncomplicated pulmonary tuberculosis. After initial hospitalization, he is now being treated at home with isoniazid, rifampin, and ethambutol. Which statement about this case is accurate? (A) A baseline auditory function test is essential before drug treatment is initiated (B) His mother, who takes care of him, does not need INH prophylaxis (C) His 3-year-old sibling should receive INH prophylaxis (D) The patient may develop symptoms of polyarthralgia caused by rifampin (E) The potential nephrotoxicity of the prescribed drugs warrants periodic assessment of renal function

7. A baseline test of ocular (not auditory) function may be useful before starting ethambutol. None of the drugs prescribed is associated with nephrotoxicity. Polyarthralgia is a common adverse effect of pyrazinamide that was not prescribed in this case. Periodic tests of liver function may be advisable in younger patients who are treated with INH plus rifampin, especially if higher doses of these drugs are used. Prophylaxis with INH is advisable for all household members and very close contacts of patients with active tuberculosis, especially young children. The answer is C.

7. Diethylstilbestrol (DES) should never be used in pregnant women because it is associated with which of the following? (A) Deep vein thrombosis (B) Feminization of the external genitalia of male offspring (C) Infertility and development of vaginal cancer in female offspring (D) Miscarriages (E) Virilization of the external genitalia of female offspring

7. Diethylstilbestrol (DES) is a nonsteroidal estrogen agonist. Several decades ago, misguided use of the drug in pregnant women appears to have resulted in fetal damage that predisposed female offspring to infertility and a rare form of vaginal cancer. For this reason, the drug should be avoided in pregnant women. Other estrogenic drugs do not appear to have these same effects. Although estrogens do increase the risk of deep vein thrombosis, this is not the reason why DES should be avoided. The answer is C.

7. A 56-year-old woman with systemic lupus erythematosus had been maintained on a moderate daily dose of prednisone for 9 mo. Her disease has finally gone into remission and she now wishes to gradually taper and then discontinue the prednisone. Gradual tapering of a glucocorticoid is required for recovery of which of the following? (A) Depressed release of insulin from pancreatic B cells (B) Hematopoiesis in the bone marrow (C) Normal osteoblast function (D) The control by vasopressin of water excretion (E) The hypothalamic-pituitary-adrenal system

7. Exogenous glucocorticoids act at the hypothalamus and pituitary to suppress the production of CRF and ACTH. As a result, adrenal production of endogenous corticosteroids is suppressed. On discontinuance, the recovery of normal hypothalamic-pituitary-adrenal function occurs slowly. Glucocorticoid doses must be tapered slowly, over several months, to prevent adrenal insufficiency. The answer is E.

7. Regarding the toxicity of aminoglycosides, which statement is accurate? (A) Gentamicin and tobramycin are the least likely to cause renal damage (B) Ototoxicity due to amikacin and gentamicin includes vestibular dysfunction that is often irreversible (C) Ototoxicity is reduced if loop diuretics are used to facilitate aminoglycoside renal excretion (D) Skin reactions are rare with use of topical neomycin (E) With traditional dosage regimens, the earliest sign of nephrotoxicity is a reduced blood creatinine

7. Gentamicin and tobramycin are the most likely aminoglycosides to cause nephrotoxicity. The incidence of nephrotoxic effects with gentamicin is 2 to 3 times greater than the incidence of ototoxicity. With traditional dosage regimens, the first indication of potential nephrotoxicity is an increase in trough serum levels of aminoglycosides, which is followed by an increase in blood creatinine. Although ototoxicity resulting from aminoglycosides such as gentamicin and tobramycin usually involves irreversible effects on vestibular function, hearing loss can also occur. Ototoxicity is enhanced by loop diuretics. Skin reactions are common with topical use of neomycin. The answer is B.

8. Which parasite is susceptible to niclosamide? (A) Ascaris lumbricoides (B) Echinococcus granulosus (C) Fasciola hepatica (D) Necator americanus (E) Taenia solium

8. Niclosamide is not active against nematodes or flukes with the exception of the large intestinal fluke. It is considered a co-drug of choice (with praziquantel) to treat common tapeworm infections because it is usually effective in a single dose. The drug is minimally absorbed from the gastrointestinal tract and causes few side effects. The answer is E.

7. A 3-year-old girl presented with hirsutism, breast enlargement, and a height and bone age that was consistent with an age of 9. Diagnostic testing revealed precocious puberty. Which of the following is the most appropriate drug for treatment of this patient's precocious puberty? (A) Atosiban (B) Follitropin (C) Leuprolide (D) Octreotide (E) Pegvisomant

7. In precocious puberty, the hypothalamic-pituitary-gonadal axis becomes prematurely active for reasons that are not understood. Treatment involves suppressing gonadotropin secretion with continuous administration of a long-acting GnRH agonist such as leuprolide. The answer is C.

Questions 6 and 7. A 28-year-old man living on the East Coast was transferred by his employer to central California for several months. On his return, he complains of having influenza-like symptoms with fever and a cough. He also has red, tender nodules on his shins. His physician suspects that these symptoms are due to coccidioidomycosis contracted during his stay in California. 7. Which is the drug of choice if this patient is suffering from persistent lung lesions or disseminated disease caused by Coccidioides immitis? (A) Amphotericin B (B) Fluconazole (C) Ketoconazole (D) Micofungin (E) Terbinafine

7. In progressive or disseminated forms of coccidioidomycosis, systemic antifungal drug treatment is needed. Until recently, amphotericin B was the recommended therapy, but fluconazole or itraconazole are now generally preferred. Note that the risk of dissemination is much greater in African Americans (10% incidence) and in pregnant women during the third trimester. The answer is B.

Questions 7-9. A 24-year-old woman comes to a clinic with complaints of dry cough, headache, fever, and malaise, which have lasted 3 or 4 d. She appears to have some respiratory difficulty, and chest examination reveals rales but no other obvious signs of pulmonary involvement. However, extensive patchy infiltrates are seen on chest x-ray film. Gram stain of expectorated sputum fails to reveal any bacterial pathogens. The patient mentions that a colleague at work has similar symptoms to those she is experiencing. The patient has no history of serious medical problems. She takes loratadine for allergies and supplementary iron tablets, and she drinks at least 6 cups of caffeinated coffee per day. The physician makes an initial diagnosis of community-acquired pneumonia. 7. Regarding the treatment of this patient, which of the following drugs is most suitable? (A) Amoxicillin (B) Clindamycin (C) Doxycycline (D) Linezolid (E) Vancomycin

7. It is often difficult to establish a definite cause of communityacquired pneumonia (CAP). Approximately 86% of cases are caused by typical pathogens such as S pneumoniae, H influenzae, or M catarrhalis, and 15% are due to the nonzoonotic atypical pathogens such as Legionella species, Mycoplasma species, or C pneumoniae. Currently, monotherapy coverage of both typical and atypical pathogens in CAP is preferred to doubledrug therapy. Preferred initial therapy includes a macrolide, doxycycline, or a quinolone active against respiratory pathogens (Chapter 46). Amoxicillin, clindamycin, and vancomycin have low activity against atypical pathogens in CAP. The answer is C.

7. Which drug causes muscle paralysis in nematodes by enhancing the actions of GABA? (A) Albendazole (B) Diethylcarbamazine (C) Ivermectin (D) Mebendazole (E) Pyrantel pamoate

7. Ivermectin and piperazine (not listed) both cause muscle paralysis in nematodes by acting through GABA receptors. Pyrantel pamoate relaxes muscles by blocking nicotinic receptors. Diethylcarbamazine also causes muscle relaxation, but the mechanism is unknown. The answer is C.

Questions 6 and 7. A male patient presents with lower abdominal discomfort, flatulence, and occasional diarrhea. A diagnosis of intestinal amebiasis is made, and E histolytica is identified in his diarrheal stools. An oral drug is prescribed, which reduces his intestinal symptoms. Later he presents with severe dysentery, right upper quadrant pain, weight loss, fever, and an enlarged liver. Amebic liver abscess is diagnosed, and the patient is hospitalized. He has a recent history of drug treatment for a tachyarrhythmia. 7. The drug regimen most likely to be effective in treating severe extraintestinal disease in this patient is (A) Chloroquine (B) Diloxanide furoate plus iodoquinol (C) Emetine plus diloxanide furoate plus chloroquine (D) Pentamidine followed by mefloquine (E) Tinidazole plus diloxanide furoate

7. Metronidazole given for 10 d, or tinidazole for 5 d, plus a luminal agent is effective in most cases of hepatic abscess, and these regimens have the dual advantage of being both amebicidal and active against anaerobic bacteria. Though active in amebic hepatic abscess, treatment with emetine is contraindicated in patients with a history of cardiac disease. The answer is E.

7. When initiating T4 therapy for an elderly patient with longstanding hypothyroidism, it is important to begin with small doses to avoid which of the following? (A) A flare-up of exophthalmos (B) Acute renal failure (C) Hemolysis (D) Overstimulation of the heart (E) Seizures

7. Patients with longstanding hypothyroidism, especially those who are elderly, are highly sensitive to the stimulatory effects of T4 on cardiac function. Administration of regular doses can cause overstimulation of the heart and cardiac collapse. The answer is D.

Questions 6-8. A 52-year-old man (weight 70 kg) is brought to the hospital emergency department in a confused and delirious state. He has had an elevated temperature for more than 24 h, during which time he had complained of a severe headache and had suffered from nausea and vomiting. Lumbar puncture reveals an elevated opening pressure, and cerebrospinal fluid findings include elevated protein, decreased glucose, and increased neutrophils. Gram stain of a smear of cerebrospinal fluid reveals gram-positive diplococci, and a preliminary diagnosis is made of purulent meningitis. The microbiology report informs you that for approximately 15% of S pneumoniae isolates in the community, the minimal inhibitory concentration for penicillin G is 20 mcg/mL. 7. Resistance of pneumococci to penicillin G is due to (A) Beta-lactamase production (B) Changes in chemical structure of target penicillinbinding proteins (C) Changes in porin structure (D) Changes in the D-Ala-D-Ala building block of peptidoglycan precursor (E) Decreased intracellular accumulation of penicillin G

7. Pneumococcal resistance to penicillins is due to changes in the chemical structures of the target penicillin-binding proteins located in the bacterial cytoplasmic membrane. A similar mechanism underlies the resistance of staphylococci to methicillin (MRSA strains). A structural alteration in the D-Ala-D-Ala component of the pentapeptide side chains of peptidoglycans is the basis for a mechanism of resistance to vancomycin. The answer is B.

7. A cancer cell that is resistant to the effects of both vincristine and methotrexate probably has developed the resistance as a result of which of the following mechanisms? (A) Changes in the properties of a target enzyme (B) Decreased activity of an activating enzyme (C) Increased expression of a P-glycoprotein transporter (D) Increased production of drug-trapping molecules (E) Increase in proteins that are involved in DNA repair

7. The P-glycoprotein family of transporters moves foreign molecules out of cells. Cancer cells acquire resistance to multiple drugs that act through different mechanisms by increasing the expressions of genes encoding these transporters. The answer is C.

7. Which adverse effect is most likely to occur with sulfonamides? (A) Fanconi's aminoaciduria syndrome (B) Hematuria (C) Kernicterus in the newborn (D) Neurologic dysfunction (E) Skin reactions

7. The most common adverse effect of the sulfonamides is a skin rash caused by hypersensitivity. Neurologic dysfunction and hematuria occur less frequently. Sulfonamides are usually avoided in the third trimester of pregnancy or in neonates, so kernicterus is rare. Fanconi's syndrome is associated with the use of outdated tetracyclines. The answer is E.

7. A 27-year-old nursing mother is diagnosed as suffering from genital herpes. She has a history of this viral infection. Previously, she responded to a drug used topically. Apart from her current problem, she is in good health. Which drug to be used orally is most likely to be prescribed at this time? (A) Amantadine (B) Foscarnet (C) Ritonavir (D) Trifluridine (E) Valacyclovir

7. Three of the drugs listed (foscarnet, trifluridine, valacyclovir) are active against strains of herpes simplex virus. Foscarnet is not used in genital infections (HSV-2) because clinical efficacy has not been established, it has poor oral bioavailability and the drug causes many toxic effects. Trifluridine is used topically but only for herpes keratoconjunctivitis (HSV-1). Valacyclovir is converted to acyclovir by first-pass metabolism in the intestine and liver. The answer is E.

Questions 6-8. A 52-year-old man (weight 70 kg) is brought to the hospital emergency department in a confused and delirious state. He has had an elevated temperature for more than 24 h, during which time he had complained of a severe headache and had suffered from nausea and vomiting. Lumbar puncture reveals an elevated opening pressure, and cerebrospinal fluid findings include elevated protein, decreased glucose, and increased neutrophils. Gram stain of a smear of cerebrospinal fluid reveals gram-positive diplococci, and a preliminary diagnosis is made of purulent meningitis. The microbiology report informs you that for approximately 15% of S pneumoniae isolates in the community, the minimal inhibitory concentration for penicillin G is 20 mcg/mL. 8. If this patient had been 82 years old and the Gram stain of the smear of cerebrospinal fluid had revealed gram-positive rods resembling diphtheroids, the antibiotic regimen for empiric treatment would include (A) Ampicillin (B) Aztreonam (C) Cefazolin (D) Fosfomycin (E) Meropenem

8. Diphtheroid-like gram-positive rods in the cerebrospinal fluid smear of an elderly patient are indicative of L monocytogenes. Listeria infections are more common in neonates, elderly patients, and those who have been treated with immunosuppressive agents. Treatment consists of ampicillin with or without an aminoglycoside such as gentamicin. Trimethoprimsulfamethoxazole can also be used (see Chapter 46). The answer is A.

8. Which statement about antitubercular drugs is accurate? (A) Antimycobacterial actions of streptomycin involve inhibition of arabinosyl transferases (B) Cross-resistance of M tuberculosis to isoniazid and pyrazinamide is common (C) Ocular toxicity of ethambutol is prevented by thiamine (D) Pyrazinamide treatment should be discontinued immediately if hyperuricemia occurs (E) Resistance to ethambutol involves mutations in the emb gene

8. Ethambutol inhibits arabinosyl transferases. Ocular toxicity due to ethambutol is dose-dependent and is usually reversible when the drug is discontinued. Thiamine is not protective. There is minimal cross-resistance between pyrazinamide and other antimycobacterial drugs. Pyrazinamide uniformly causes hyperuricemia, but this is not a reason to halt therapy even though the drug may provoke gouty arthritis in susceptible persons. The answer is E.

8. After a backpacking trip in the mountains, a 24-year-old man develops diarrhea. He acknowledges drinking stream water without purification, and you suspect he is showing symptoms of giardiasis. Because you know that laboratory detection of cysts or trophozoites in the feces can be difficult, you decide to treat the patient empirically with (A) Chloroquine (B) Emetine (C) Metronidazole (D) Pentamidine (E) TMP-SMZ

8. Giardiasis is a common intestinal protozoan infection caused by Giardia lamblia. A large number of infections result from fecal contamination of food or water. Metronidazole has been considered the drug of choice. Tinidazole is equally effective. The answer is C.

8. Which drug is least likely to be effective in the treatment of esophageal candidiasis if it is used by the oral route? (A) Clotrimazole (B) Fluconazole (C) Griseofulvin (D) Itraconazole (E) Nystatin

8. Griseofulvin has no activity against C albicans and is not effective in the treatment of systemic or superficial infections caused by such organisms. "Swish and swallow" formulations of clotrimazole and nystatin have been used commonly. Most of the azoles are effective in esophageal candidiasis. The answer is C.

Questions 8 and 9. A 54-year-old man with advanced tuberculosis has developed signs of severe acute adrenal insufficiency. 8. Which of the following signs or symptoms is this patient most likely to exhibit? (A) A moon face (B) Dehydration (C) Hyperglycemia (D) Hypertension (E) Hyperthermia

8. In acute adrenal insufficiency, there is loss of salt and water that is primarily due to reduced production of aldosterone. The loss of salt and water can lead to dehydration. The answer is B.

8. A 47-year-old man exhibited signs and symptoms of acromegaly. Radiologic studies indicated the presence of a large pituitary tumor. Surgical treatment of the tumor was only partially effective in controlling his disease. At this point, which of the following drugs is most likely to be used as pharmacologic therapy? (A) Cosyntropin (B) Desmopressin (C) Leuprolide (D) Octreotide (E) Somatropin

8. Octreotide, a somatostatin analog, has some efficacy in reducing the excess GH production that causes acromegaly. The answer is D.

8. Which of the following is a unique property of SERMs? (A) Act as agonists in some tissues and antagonists in other tissues (B) Activate a unique plasma membrane-bound receptor (C) Have both estrogenic and progestational agonist activity (D) Inhibit the aromatase enzyme required for estrogen synthesis (E) Produce estrogenic effects without binding to estrogen receptors

8. SERMs such as tamoxifen and raloxifene exhibit tissuespecific estrogenic and antiestrogenic effects. The answer is A.

8. Which drug is effective in the treatment of nocardiosis and, in combination with pyrimethamine, is prophylactic against Pneumocystis jiroveci infections in AIDS patients? (A) Amoxicillin (B) Ciprofloxacin (C) Clindamycin (D) Sulfadiazine (E) Trimethoprim

8. Sulfadiazine and TMP-SMZ are drugs of choice in nocardiosis. In combination with pyrimethamine (an effective dihydrofolate reductase inhibitor in protozoa), sulfadiazine is effective in toxoplasmosis and is prophylactic against pneumocystis pneumonia in the AIDS patient. However, TMP-SMZ is more commonly used for the latter purpose. The answer is D.

8. Hormone produced in the peripheral tissues when T4 is administered (A) 131I (B) Amiodarone (C) Propranolol (D) Propylthiouracil (E) Triiodothyronine

8. T4 is converted into T3 in the periphery. The answer is E.

Questions 8 and 9. A 23-year-old man with Hodgkin's lymphoma was treated unsuccessfully with the MOPP regimen (mechlorethamine, vincristine, prednisone, procarbazine). He subsequently underwent a successful course of therapy with the ABVD regimen (doxorubicin, bleomycin, vinblastine, dacarbazine). 8. Which of the following classes of anticancer drugs used in the treatment of this patient is cell cycle specific (CCS) and used in both the MOPP and ABVD regimens? (A) Alkylating agents (B) Antibiotics (C) Antimetabolites (D) Glucocorticoids (E) Plant alkaloids

8. The cell cycle-specific drugs used in standard treatment protocols for Hodgkin's lymphoma are bleomycin and the vinca alkaloids. Vinblastine is used in the ABVD regimen, and vincristine (Oncovin) is used in the MOPP regimen. The answer is E.

Questions 7-9. A 24-year-old woman comes to a clinic with complaints of dry cough, headache, fever, and malaise, which have lasted 3 or 4 d. She appears to have some respiratory difficulty, and chest examination reveals rales but no other obvious signs of pulmonary involvement. However, extensive patchy infiltrates are seen on chest x-ray film. Gram stain of expectorated sputum fails to reveal any bacterial pathogens. The patient mentions that a colleague at work has similar symptoms to those she is experiencing. The patient has no history of serious medical problems. She takes loratadine for allergies and supplementary iron tablets, and she drinks at least 6 cups of caffeinated coffee per day. The physician makes an initial diagnosis of community-acquired pneumonia. 8. If this patient were to be treated with the macrolide erythromycin, she should (A) Avoid exposure to sunlight (B) Avoid taking supplementary iron tablets (C) Decrease her intake of caffeinated beverages (D) Discontinue loratadine temporarily (E) Have her plasma urea nitrogen or creatinine checked before treatment

8. The inhibition of liver cytochrome P450 by erythromycin has led to serious drug interactions. Although erythromycin does not inhibit loratadine metabolism, it does inhibit the CYP1A2 form of cytochrome P450, which metabolizes methylxanthines. Consequently, cardiac and/or CNS toxicity may occur with excessive ingestion of caffeine. Unlike the tetracyclines, the oral absorption of erythromycin is not affected by cations and the drug does not cause photosensitivity. Because erythromycin undergoes biliary excretion, there is little reason to assess renal function before treatment. The answer is C.

8. Oral formulations of this drug should not be used in a pregnant AIDS patient because they contain propylene glycol. One of the characteristic adverse effects of the drug is hyperpigmentation on the palms of the hands and soles of the feet especially in African-American patients. (A) Amprenavir (B) Efavirenz (C) Emtricitabine (D) Enfuvirtide (E) Zalcitabine

8. Three of the drugs listed should be avoided, or used with extreme caution, in the pregnant patient. Oral forms of amprenavir and emtricitabine both contain propylene glycol, a potentially toxic compound. Efavirenz has caused fetal abnormalities in pregnant monkeys. However, one of the distinctive adverse effects of emtricitabine is hyperpigmentation. The answer is C.

8. This drug has characteristics almost identical to those of gentamicin but has much weaker activity in combination with penicillin against enterococci. (A) Amikacin (B) Erythromycin (C) Netilmicin (D) Spectinomycin (E) Tobramycin

8. Tobramycin is almost identical to gentamicin in both its pharmacodynamic and pharmacokinetic properties. However, it is much less active than either gentamicin or streptomycin when used in combination with a penicillin in the treatment of enterococcal endocarditis. The answer is E.

Questions 8 and 9. A 54-year-old man with advanced tuberculosis has developed signs of severe acute adrenal insufficiency. 9. The patient should be treated immediately. Which of the following combinations is most rational? (A) Aldosterone and fludrocortisone (B) Cortisol and fludrocortisone (C) Dexamethasone and metyrapone (D) Fludrocortisone and metyrapone (E) Triamcinolone and dexamethasone

9. A rational combination of drugs should include agents with complementary effects (ie, a glucocorticoid and a mineralocorticoid). The combination with these characteristics is cortisol and fludrocortisone. (Note that although fludrocortisone may have sufficient glucocorticoid activity for a patient with mild disease, a patient in severe acute adrenal insufficiency needs a full glucocorticoid such as cortisol.) The answer is B.

9. Your 23-year-old female patient is pregnant and has gonorrhea. The medical history includes anaphylaxis following exposure to amoxicillin. The most appropriate drug to use is (A) Azithromycin (B) Cefixime (C) Ceftriaxone (D) Ciprofloxacin (E) Doxycycline

9. All of the listed drugs have been used for treatment of gonorrhea. Cephalosporins should be avoided in patients with a history of severe hypersensitivity to penicillins, and fluoroquinolones (see Chapter 46) should be avoided in pregnancy. Tetracyclines including doxycycline have been used in the past for gonorrhea, but not as single doses, and they too should be avoided in pregnancy. The answer is A.

9. Antiarrhythmic drug that inhibits peripheral conversion of T4 to T3 (A) 131I (B) Amiodarone (C) Propranolol (D) Propylthiouracil (E) Triiodothyronine

9. Amiodarone is an iodine-containing antiarrhythmic drug with complex effects on the thyroid gland and thyroid hormones. One of its actions is to inhibit peripheral conversion of T4 to T3. The answer is B.

Questions 7-9. A 24-year-old woman comes to a clinic with complaints of dry cough, headache, fever, and malaise, which have lasted 3 or 4 d. She appears to have some respiratory difficulty, and chest examination reveals rales but no other obvious signs of pulmonary involvement. However, extensive patchy infiltrates are seen on chest x-ray film. Gram stain of expectorated sputum fails to reveal any bacterial pathogens. The patient mentions that a colleague at work has similar symptoms to those she is experiencing. The patient has no history of serious medical problems. She takes loratadine for allergies and supplementary iron tablets, and she drinks at least 6 cups of caffeinated coffee per day. The physician makes an initial diagnosis of community-acquired pneumonia. 9. A 5-d course of treatment for community-acquired pneumonia would be effective in this patient with little risk of drug interactions if the drug prescribed were (A) Ampicillin (B) Azithromycin (C) Clindamycin (D) Erythromycin (E) Vancomycin

9. Azithromycin has a half-life of more than 70 h, which allows for once-daily dosing and a 5-d course of treatment for community-acquired pneumonia. Unlike other macrolides, azithromycin does not inhibit cytochrome P450 enzymes involved in drug metabolism. The answer is B.

9. Once weekly administration of which of the following antibiotics has prophylactic activity against bacteremia caused by M avium complex in AIDS patients? (A) Azithromycin (B) Clarithromycin (C) Ethambutol (D) Kanamycin (E) Rifabutin

9. Because of its long elimination half-life (3-4 d), weekly administration of azithromycin has proved to be equivalent to daily administration of clarithromycin when used for prophylaxis against M avium complex in AIDS patients. The answer is A.

9. Which statement about ciprofloxacin is accurate? (A) Active against most MRSA strains of staphylococci (B) Antagonism occurs if it is used with inhibitors of dihydrofolate reductase (C) During treatment, tendinitis and even tendon rupture may occur (D) Most "first-time" urinary tract infections are resistant to ciprofloxacin (E) Organisms associated with middle ear infections are highly resistant

9. Ciprofloxacin is commonly used for the treatment of urinary tract infections and is active against most strains of common causative agents of otitis media, including H influenzae and pneumococci. However, up to 50% of strains of MRSA are now resistant to ciprofloxacin. No clinical antagonism has been reported between fluoroquinolones and inhibitors of folic acid synthesis. Fluoroquinolones are not recommended for use in pregnancy or for children younger than 10 years because they may damage growing cartilage. The answer is C.

9. Which adverse effect occurs with the use of mebendazole during intestinal nematode therapy? (A) Cholestatic jaundice (B) Corneal opacities (C) Hirsutism (D) Peripheral neuropathy (E) None of the above

9. Doses of mebendazole required for intestinal nematode therapy are almost free of adverse effects even in the malnourished or debilitated patient. Gastrointestinal distress may occur in children with ascariasis who are heavily parasitized, together with a slight headache or dizziness. Avoid the drug in children under 2 yr of age because of rare reports of seizures. The answer is E.

9. Finasteride has efficacy in the prevention of male-pattern baldness by virtue of its ability to do which of the following? (A) Competitively antagonize androgen receptors (B) Decrease the release of gonadotropins (C) Increase the serum concentration of sex hormonebinding globulin (D) Inhibit the synthesis of testosterone (E) Reduce the production of dihydrotestosterone

9. Finasteride inhibits 5α-reductase, the enzyme that converts testosterone to DHT, the principal androgen in androgensensitive hair follicles. The answer is E.

9. A patient needs antibiotic treatment for native valve, culturepositive infective enterococcal endocarditis. His medical history includes a severe anaphylactic reaction to penicillin G during the last year. The best approach would be treatment with (A) Amoxicillin-clavulanate (B) Aztreonam (C) Cefazolin plus gentamicin (D) Meropenem (E) Vancomycin

9. In patients who have had a severe reaction to a penicillin, it is inadvisable to administer a cephalosporin or a carbapenem such as meropenem. Aztreonam has no significant activity against gram-positive cocci, so the logical treatment in this case is vancomycin, often with an aminoglycoside (eg, gentamicin) for synergistic activity against enterococci. The answer is E.

9. This drug can clear trypanosomes from the blood and lymph nodes and is active in the late CNS stages of African sleeping sickness. (A) Emetine (B) Eflornithine (C) Melarsoprol (D) Nifurtimox (E) Suramin

9. In the advanced stages of African sleeping sickness, melarsoprol is the drug of choice because, unlike pentamidine or suramin, it effectively enters the CNS. Nifurtimox is the most commonly used drug for Chagas' disease. The answer is C.

9. Serious cardiac effects have occurred when this drug was taken by patients using the antihistamines astemizole or terfenadine (A) Amphotericin B (B) Fluconazole (C) Griseofulvin (D) Ketoconazole (E) Terbinafine

9. Ketoconazole was the first oral azole introduced into clinical use, but it has a greater propensity to inhibit human cytochrome P450 enzymes than other azoles and is no longer widely used in the United States. Cardiotoxicity may occur when ketoconazole is used by patients taking astemizole or terfenadine as a result of the ability of ketoconazole to inhibit their metabolism via hepatic cytochromes P450. The answer is D.

9. Regarding interferon-α, which of the following statements is false? (A) At the start of treatment, most patients experience flu-like symptoms (B) Indications include treatment of genital warts (C) It is used in the management of hepatitis B and C (D) Lamivudine interferes with its activity against hepatitis B (E) Toxicity includes bone marrow suppression

9. Lamivudine is used in monotherapy of HBV infections and does not oppose the beneficial effects of interferon-α when both agents are used together in the treatment of hepatitis B. The answer is D.

9. A 37-year-old woman with infertility due to obstructed fallopian tubes was undergoing ovulation induction in preparation for in vitro fertilization. After 10 d of treatment with leuprolide, the next step in the procedure is most likely to involve 10-14 d of treatment with which of the following? (A) Bromocriptine (B) Follitropin (C) Gonadorelin (D) hCG (E) Pergolide

9. Once the patient's endogenous gonadotropin production has been inhibited through continuous administration of the GnRH agonist leuprolide, the next step in ovulation induction is the administration of a drug with FSH activity to stimulate follicle maturation. Follitropin is recombinant FSH. The only other drug listed that is used in ovulation induction is hCG, but this is an LH analog. The answer is B.

Questions 8 and 9. A 23-year-old man with Hodgkin's lymphoma was treated unsuccessfully with the MOPP regimen (mechlorethamine, vincristine, prednisone, procarbazine). He subsequently underwent a successful course of therapy with the ABVD regimen (doxorubicin, bleomycin, vinblastine, dacarbazine). 9. During the second course of drug treatment (ABVD regimen), this patient developed dyspnea, a nonproductive cough, and intermittent fever. Chest x-ray film revealed pulmonary infiltration. If these problems are due to the anticancer drugs to which he has been exposed, which of the following is the most likely causative agent? (A) Bleomycin (B) Dacarbazine (C) Doxorubicin (D) Prednisone (E) Vinblastine

9. The anticancer drug most commonly associated with pulmonary toxicity is bleomycin. The answer is A.

Questions 1 and 2. A 13-year-old boy with type 1 diabetes is brought to the hospital complaining of dizziness. Laboratory findings include severe hyperglycemia, ketoacidosis, and a blood pH of 7.15. 2. Which of the following is the most likely complication of insulin therapy in this patient? (A) Dilutional hyponatremia (B) Hypoglycemia (C) Increased bleeding tendency (D) Pancreatitis (E) Severe hypertension

Because of the risk of brain damage, the most important complication of insulin therapy is hypoglycemia. The other choices are not common effects of insulin. The answer is B.

6. A 54-year-old obese patient with type 2 diabetes has a history of alcoholism. In this patient, metformin should either be avoided or used with extreme caution because the combination of metformin and ethanol increases the risk of which of the following? (A) A disulfiram-like reaction (B) Excessive weight gain (C) Hypoglycemia (D) Lactic acidosis (E) Serious hepatotoxicity

Biguanides, especially the older drug phenformin, have been associated with lactic acidosis. Thus, metformin should be avoided or used with extreme caution in patients with conditions that increase the risk of lactic acidosis, including acute ethanol ingestion. The answer is D.

4. Which one of the following drugs promotes the release of endogenous insulin? (A) Acarbose (B) Glipizide (C) Metformin (D) Miglitol (E) Pioglitazone

Glipizide is a second-generation sulfonylurea that promotes insulin release by closing potassium channels in pancreatic B cells. The answer is B.

10. Which of the following patients is most likely to be treated with intravenous glucagon? (A) An 18-year-old woman who took an overdose of cocaine and now has a blood pressure of 190/110 mm Hg (B) A 27-year-old woman with severe diarrhea caused by a flare in her inflammatory bowel disease (C) A 57-year-old woman with type 2 diabetes who has not taken her glyburide for the last 3 d (D) A 62-year-old man with severe bradycardia and hypotension resulting from ingestion of an overdose of atenolol (E) A 74-year-old man with lactic acidosis as a complication of severe infection and shock

Glucagon acts through cardiac glucagon receptors to stimulate the rate and force of contraction of the heart. Because this bypasses cardiac β adrenoceptors, glucagon is useful in the treatment of β-blocker-induced cardiac depression. The answer is D.

5. Which of the following is an important effect of insulin? (A) Increased conversion of amino acids into glucose (B) Increased gluconeogenesis (C) Increased glucose transport into cells (D) Inhibition of lipoprotein lipase (E) Stimulation of glycogenolysis

Insulin lowers serum glucose concentration in part by driving glucose into cells, particularly into muscle cells. The answer is C.

3. A 24-year-old woman with type 1 diabetes wishes to try tight control of her diabetes to improve her long-term prognosis. Which of the following regimens is most appropriate? (A) Morning injections of mixed insulin lispro and insulin aspart (B) Evening injections of mixed regular insulin and insulin glargine (C) Morning and evening injections of regular insulin, supplemented by small amounts of NPH insulin at mealtimes (D) Morning injections of insulin glargine, supplemented by small amounts of insulin lispro at mealtimes (E) Morning injection of NPH insulin and evening injection of regular insulin

Insulin regimens for tight control usually take the form of establishing a basal level of insulin with a small amount of a long-acting preparation (eg, insulin glargine) and supplementing the insulin levels, when called for by food intake, with short-acting insulin lispro. Less tight control may be achieved with 2 injections of intermediate-acting insulin per day. Because intake of glucose is mainly during the day, longacting insulins are usually given in the morning, not at night. The answer is D.

Questions 1 and 2. A 13-year-old boy with type 1 diabetes is brought to the hospital complaining of dizziness. Laboratory findings include severe hyperglycemia, ketoacidosis, and a blood pH of 7.15. 1. Which of the following agents should be administered to achieve rapid control of the severe ketoacidosis in this diabetic boy? (A) Crystalline zinc insulin (B) Glyburide (C) Insulin glargine (D) NPH insulin (E) Tolbutamide

Oral antidiabetic agents (listed in Table 41-1) are inappropriate in this patient because he has insulin-dependent diabetes. He needs a rapid-acting insulin preparation that can be given intravenously (see Figure 41-1). The answer is A.

8. The PPAR-γ receptor that is activated by thiazolidinediones increases tissue sensitivity to insulin by which of the following mechanisms? (A) Activating adenylyl cyclase and increasing the intracellular concentration of cAMP (B) Inactivating a cellular inhibitor of the GLUT2 glucose transporter (C) Inhibiting acid glucosidase, a key enzyme in glycogen breakdown pathways (D) Regulating transcription of genes involved in glucose utilization (E) Stimulating the activity of a tyrosine kinase that phosphorylates the insulin receptor

The PPAR-γ receptor belongs to a family of nuclear receptors. When activated, these receptors translocate to the nucleus, where they regulate the transcription of genes encoding proteins involved in the metabolism of carbohydrate and lipids. The answer is D.

9. Which of the following drugs is most likely to cause hypoglycemia when used as monotherapy in the treatment of type 2 diabetes? (A) Acarbose (B) Glyburide (C) Metformin (D) Miglitol (E) Rosiglitazone

The insulin secretagogues, including the sulfonylurea glyburide, can cause hypoglycemia as a result of their ability to increase serum insulin levels. The biguanides, thiazolidinediones, and α-glucosidase inhibitors are euglycemics that are unlikely to cause hypoglycemia when used alone. The answer is B.

7. Which of the following drugs is taken during the first part of a meal for the purpose of delaying the absorption of dietary carbohydrates? (A) Acarbose (B) Exenatide (C) Glipizide (D) Pioglitazone (E) Repaglinide

To be absorbed, carbohydrates must be converted into monosaccharides by the action of α-glucosidase enzymes in the gastrointestinal tract. Acarbose inhibits α-glucosidase and, when present during digestion, delays the uptake of carbohydrates. The answer is A.


Ensembles d'études connexes

Section 16: Real Estate Financing: Mortgage Loans

View Set

Final Quarter 11-14 Intro Psychology

View Set

Chapter 9: Teaching and Counseling wk 5

View Set

2nd Semester Exam: Anatomy Regulars

View Set

NC earth/enviromental science exam released test

View Set